1st series [1] [2] [3] [4] [5] [6] [7] [8] [9] [10] [11] [12] [13] [14] [15] [16] [17] [18] [19] [20] [21] [22] [23] [24]  2nd series [1] [2] [3] [4] [5] [6] [7] [8] [9] [10] [11] [12] [13] [14] [15] [16] [17] [18] [19] [20] [21] [22] [23] [24] [25] [26] [27] [28] [29] [30] [31] [32] [33] [34] [35] [36] [37] [38] [39] [40] [41] [42] [43] [44] [45] [46] [47] [48] [49]

  View the latest questions and answers at askaphilosopher.org

Ask a Philosopher: Questions and Answers 21 (2nd series)

When referring to an answer on this page, please quote the page number followed by the answer number. The first answer on this page is 21/1.

(1) Steve asked:

If a child were brought up in a home of crawling adults and were never exposed to walking man would that child walk as a natural instinct? Birds fly would man walk?

---

Well this is an experiment we can never perform so we can only answer it by guessing based on animal studies etc. My guess is that the child would learn to walk but wouldn't find walking useful since presumably house would have very low ceilings so that the crawling adults could reach the light switches and change the lightbulbs. So it would be impossible to stand up inside a house once you were beyond a certain age. Fridges would also be very small and in general our human way of life would be impossible. Imagine crawling to the railway station.

Shaun Williamson

back

(2) Dasha asked:

If a person tells you to put your hands on your mouth how do you know where to put them? Well because your parents told you and their parents told them but if we evolved from monkeys then how did the first man know where his/her mouth was if he/she didn't know. No one would have him/her.

---

Well if you ask a baby to put its hands on your mouth it won't do it because it doesn't talk and doesn't understand language yet. This has nothing to do with what your parents tell you. The idea of the first man is misleading. We didn't evolve from monkeys but from an earlier common ancestor. Language and the use of language also evolved very slowly. The theory of evolution is a complex one and at the moment you don't understand it. If you did then you would understand that you are asking the wrong questions.

Shaun Williamson

back

(3) Dasha asked:

What comes first the egg or the chicken?

---

You need to study the theory of evolution and then you will come to understand that your question is not worth asking.

Shaun Williamson

back

(4) Armilah asked:

How would you describe the color red and green to a blind child since birth?

---

I wouldn't even try to do this. What could it achieve. I remember someone who once asked a man who had been blind from birth. What's it like being blind? How was he supposed to answer this?

Shaun Williamson

back

(5) Steve asked:

I'm not sure if this is even a philosophy question, probably more of a juvenile sixth form angst question, but my question is how do you unthink, how do you unbecome the person that you are? I seem to have approx the same words and phrases going round and round in my head, namely. 'what am I doing with my life? I feel so flat and miserable and sensitive and disinterested, what's the point?' it's more than just depression. we are going to get old and die, it's inevitable. it all seems pointless. what's the point of doing stuff when all it is is to prevent you from doing nothing? when you derive no pleasure from anything because you are still aware that you exist and feel socially awkward and I know you have to make an effort but the thoughts don't go away — I don't want to be here, I don't want to be doing this saying this...do other people feel like this or do they just 'get on with it?' how? I have these similar thoughts a lot and don't feel I have the energy to apply them or change. I can't imagine a future where I look back and say remember when I used to have those thoughts, think and feel these things but now I don't anymore. happiness is doing not thinking. will it be like this for ever or shall I just start believing in god. is that the only thing that makes it alright? even the supposed best thing in my life, ie my daughter. even when I'm with her I still think I don't want to be here, doing this, it's all too much effort, what's the point, I wish I was dead. how do I unthink? unbecome? yes I'm self obsessed, yes I'm negative and miserable. is this it for the next fifty years? pass me the pills now please, somebody...

---

Steve there are lots of people who feel the same way as you do. You are suffering from what is called depression. The fact that you have a daughter means that your depression is not sixth form angst. I am sure that when your daughter grows up she would like you to be there. Life is very difficult for everybody but people don't often talk about these things. As to how you overcome these things, start by waking up every day and saying to yourself I am suffering from depression and everything looks black to me and life doesn't seem worth living.

Try to get medical help. Some of the pills do work. But in the end you only get one life. Not all of it is easy or happy but things can get better so stay around and see how it works out. Think about what you could do to make your life better and do it. Your life will not be the same for the next fifty years. You will be happy again.

Shaun Williamson

back

(6) Nicole asked:

Can you give proof that God does or does not exist?

---

Can you prove or disprove the existence of fairies in your garden? Just because every time you go to look you don't see them won't do it, right? So then, how do you do it?

Steven Ravett Brown

back

(7) Margaret asked:

"The day that a priest can show me how to transmogrify water into wine I will unlapse immediately." Any comments?

---

Sounds silly to me. Do you think that would constitute proof of the existence of the Catholic version of god? What about the psychic powers of the priest? What about other gods? What about sleight of hand?

Steven Ravett Brown

back

(8) Meg asked:

How would Aristotle view same sex marriages/civil unions?

---

He'd probably be a bit puzzled over 1) our conception of marriage, since for him it would basically have been a contract between a man and a woman involving children, money, property, inheritance, and so forth... not involving "love", as we are taught to think of it; 2) our fuss over classifying people as "homosexual" vs "heterosexual" vs ... whatever. For him, and as far as we know at this point in time, other ancient Greeks, the best kind of sex was between a man and another man or boy, but that didn't mean either were "homosexual", because men married and had children also. The important differences for the ancient Greeks weren't between sexual preferences, as we are taught to think they are; they were, as far as we know, between sexual functions in society.

Steven Ravett Brown

back

(9) Giulia asked:

If it is in the long run that, somehow, truth may survive through the decay of untruth: to what extent is this the case in history?

---

What do you call "truth"? There are several quite different meanings of truth. Read my essay on this in Philosophy Pathways Issue number 98 as of 7th February 2005.

There seems to be some general agreement that in mathematics and logics and in the natural sciences the idea "that, somehow, truth may survive through the decay of untruth" is at least a meaningful one. Why? Because logic, math, and nature are what they are, so we may define "truth" as "the real state of affairs" and get nearer to a true understanding of this state over the years. But even in this case, as Thomas Kuhn has shown for physics (while every historian of sciences has known it before), progress in understanding is not generally straightforward but proceeds by a chain of "paradigm shifts" separated by "normal science". See http://www.2think.org/kuhn.shtml for details. Those paradigm shifts may be compared to the genesis of new species and the extinction of others in biological evolution.

The essential precondition of scientific and technical progress is, as I hinted above, the very fact that the laws of nature are what they are. But this is not the case in interpersonal relations or in our understanding of our life and our goals and obligations etc.. If you have to find out on the nature of the genes or of the planet Jupiter, you know how to do some research on "facts" and apply some theories. But if you have to find out on the nature of your relations to other persons or to god or to your past or to your future, etc., how do you define a "research project" on these questions? You see the problem: There is no answer to those personal questions "out there". You have not to find the answer by some microscope or telescope, but you have to INVENT some answer like an artist. You have to make up your mind and to INTERPRET and DEFINE your relationships. Thus what you are confronted with is your FREEDOM. If your relation to some person is "bad", this is not by some "natural law" but may be changed by yourself and the other person working on this relation. Likewise with your past and future, which are both open to interpretation. If you see your past or future bright or gloomy this is not "in the facts" but is an artifact of your interpretation. Whether you see the glass half-full or half-empty is not a matter of the glass filled to 50% but is a matter of your evaluation.

The same applies with human "progress": Of course we all are gaining some new experience and insight all the time, but some people interpret this as a progress, while others see the same developments as a decline. The very notion of "progress" is meant to be a positive one as opposed to "decline". But these notions are evaluative and by this cannot be scientific, because science is by definition value-free. Science is only asking for facts and relations between facts, never evaluating. If you evaluate something, the question will always be "valuable for whom and by what argument?" Thus, whether you call history a progress or a decline does not depend on the historical facts, but depends on your ideas of what is good for mankind or for nature etc..

Thus your question whether "truth may survive through the decay of untruth" cannot be answered definitely, because it is seldom clear what "truth" should be in the first line. Since you are the artist of your own life — and mankind is in this same sense the artist of the history of mankind — you always may redefine the meaning of "the truth of my existence". Many people, after a religious conversion, called their former life "a life of darkness and error". Thus in their opinion some un-truth has decayed in the light of a new truth. But this remains a subjective evaluation and is not provable in any scientific way.

And what would you call truth and untruth in the arts? Surely Picasso is different from Rembrandt or Leonardo, but is any of them "more true"? By what standard? Likewise in music and literature and architecture.

There seem to be some things that we find "childish" after some time. You cannot and will not behave or feel like you are 7 when in fact you are 17 or 27 or 77. In this sense your question may have some meaning. But even then the meaning changes a bit. In what sense is your way of seeing life and the world around you more "true" when you see it with 17 than when you saw it with 7 years? It is different, even "more advanced", but to be "more true" does not mean the same as to be different or "more advanced".

This too applies with humanity: Today we cannot see slavery as something you can have or not. We think it is outdated and definitely bad and unacceptable. But we know that neither Socrates nor Plato nor Aristotle, nor even Jesus or Paulus thought this way. It is much a historical thing. Likewise duelling or suicide "for honor" or the lower status of women do not seem "natural" this time. Thus a whole frame of reference for evaluations has shifted over the centuries. But this is similar to modes of painting: Today no serious artist would engage in painting naked saints or Greek goddesses. Those times are gone. But would we call this "a decay of un-truth"? We instead have porno-pin-ups today. It's more a matter of customs than of truth.

So you cannot replace untruth by truth if you don't know what the meaning of truth could be. We are gaining experience and some insight, but truth is an evasive concept. And I even think that this is good: Life is a challenge, but not for the sciences but for the arts. See yourself as the creative artist leading your life like a movies-director leads the actions of the actors and actresses on the set. And see your life and that of mankind as a drama with a script to be always invented anew. On this see http://www.kirjasto.sci.fi/pirandel.htm and http://www.amazon.com/exec/obidos/tg/detail/-/B00006CXGW/102-2137856-4669723?v=glance and http://www.eldritchpress.org/lp/sixp.htm. Have fun!

Hubertus Fremerey

back

(10) Alex asked:

I'm debating free will against compatibilism (as believed by Daniel Dennett). At the base of his argument is his belief in event-event causation (he was probably inspired by Hume). How can event-event causation lead to an Agent (humans) with the ability to MAKE an action (have free will)? For this to happen one needs a different view of causation which is a relationship between an entity and its action, as opposed to a relationship between an event and another.

---

Well, since the concept of "compatibilism" was not known to me before, I just entered Compatibilism+Dennett in Google. Now have a look into some of this stuff yourself to find out on your question.

But of course this would be no answer, so let me do a try. What do you call an "event". If you bang your head against a wall, you will get a bump. This is a classical "event-event" reaction. But if you read a TV-program to know what movies or other information and entertainment you may have in the evening, this is somewhat more complicated: If the program happens to be written in Chinese and you don't read Chinese, then it will contain no information at all save some pictures and numbers. Thus to make this reading an "event" you first should be able to read. And then of course you should be able to understand what is written, because if your favourite movie or actor/actress or some other stuff of great interest to you is showing up at 9 pm, you may change your plans for the evening. Thus you once more need some information to make this an "event".

Now you see what it all comes to: The "event" in this case is not the written text on the magazine-paper but the event is in your brain, in the knowledge available in your memory. Without this memory-processing there would be no "event" — save the irrelevant fact that you are leafing through some printed pages of pulp.

You now have your "different view of causation which is a relationship between an entity and its action." The "entity" is you, the "relationship" is linked in your brain between "information" and "action depending on this information". The important thing to note is: The "event" (i.e. the information contained in the TV-mag) is NOT causal in any simple way for your actions. All you know about the world and about good movies and actors and about alternative ways of spending this evening etc. has been assembled in your brain more or less long since, even from the moment of your birth or before. What reading the TV-mag does is only to switch on or off some switches to make you go this way and not that one, but the "event" does not CREATE the switches or the way they are working, it only makes use of them.

And in the same way as your text-software is managed by an operating system (OS) on your computer, and the OS is managed by some firmware implemented in the booter of your PC, so your evaluating schemes in your brain and memory are organized in complex vertical and horizontal organizations like the government of a state. There seldom is simply "cause and effect". There is much "chaos" and "chance" involved. Even in your brain there is some "butterfly effect" (look it up).

Seen in this way to call human behaviour "determined by the neurons" is not wrong but is "trivial nonsense" in the same way as to say that the Moonshine-Sonata is made from hitting the right key on the piano in the right moment. This too would not be "wrong", but would be "trivial nonsense".

We are acting in a meaningful world — mostly. The neurologists are right on that you can convince a person in trance to do some absurd thing afterwards, and after awaking from the trance the person does (while not always!) what it is told and finds even some absurd explanation for what he/she does. But this is a very artificial situation. Normally we are not guided by some hypnotizer but by "situations" and by our understanding of those. Through all our years of life we have learned from elders and peers and from talks and books and movies and from our own experiences and consideration what to make of this or that situation. Thus we are "pre-programmed" in part by "phylo-genesis" (our "apish ancestry" as mammals and primates) and by "onto-genesis" (our private set of genes and our personal history and experiences) to react in a way to "events" that nobody can predict with certainty — not even we ourselves. But most of what we do becomes "meaningful" because of this complicated web of interacting cultural and personal "programs". Thus the idea that we are "determined by our neurons" is misleading at best. I have read some arguments of the best of todays neurologists, and they did not convince me that they know what they are speaking of. They argue like piano-makers trying to explain "how the Moonshine-Sonata is functioning". They are just missing the point.

I think this whole "neo-deterministic" arguing is a new form of "psychologism". Of course we need a brain ("neurons") to handle language. And there may be even a genetically determined "deep structure" of a universal human grammar in the sense suggested by Noam Chomsky. But a grammar and a language and the good use of a language are not to be analyzed by neurologists but by linguists and literary critics etc.. Thus to say that all human activity has its base in the "wet-ware" of neural tissue is a banality that does not explain much. It surely does not render human activities "meaningless". Our languages and literatures and music and arts etc. etc. remain as meaningful as before, and this is brought about by the complicated webs of culture, of collective and private thinking and arguing and experiencing. Thus don't get scared by those neurologists who seem to tell you otherwise.

And now I will have a first look into those Google-links to Dennett and his theory of compatibilism. And you should do likewise.

Hubertus Fremerey

back

(11) Alex asked:

I'm debating free will against compatibilism (as believed by Daniel Dennett). At the base of his argument is his belief in event-event causation (he was probably inspired by Hume). How can event-event causation lead to an Agent (humans) with the ability to MAKE an action (have free will)? For this to happen one needs a different view of causation which is a relationship between an entity and its action, as opposed to a relationship between an event and another.

---

Well, I'm not aware the Dennett holds to anything like a traditional conception of free will, which latter seems to boil down to being able to violate causality. I'm not sure why "event-event causation" eliminates free will and "entity-action causation" does not, since from the little you say above I see no difference between an "event" and an "action". But whatever... aside from that, Dennett's conception, as I understand it, has much more to do with one's feeling of an action being free rather than some metaphysical conception of freedom, and in fact my understanding of Dennett is that he does not in fact hold with free will in that latter sense.

Steven Ravett Brown

back

(12) Ryan asked:

I find this website very interesting, you've done a good job with it. I was asked this question for a school seminar "are there some things we can never know?"

---

"Does the Christian god exist?" is one I'd include, with the caveat that we might have to extend this to all past and future conceptions of gods that humans have created, depending on just how observable some particular god is hypothesized as being. To put it another way, when you say "things", you are opening a very nasty metaphysical can of worms. Think about it... just what is a "thing"?

Steven Ravett Brown

back

(13) Cliff asked:

Is there any philosophical concept or calculus that can be used to distinguish between contingent, necessary and sufficient attributes of an object or activity? From what I have read, this is an extremely subjective exercise, whether one is dealing with structural, functional or behavioural aspects. As an — Irish — engineer, that will not do me, at all, at all. Take these two items 'X' and 'Y' (instances of class 'x' and 'y') and as different from one another as chalk is from cheese. How do I go about establishing and prioritising their attributes — from first principles?

---

Why, as an engineer, do you want to know this? At any rate, I know of no general way to do this outside of being a fan of some particular philosopher and following their system, or of using some formal system which builds these properties in. The former is what you're objecting to, and quite rightly, I think; while the latter is, to my thinking, merely game-playing. So... you're stuck. My advice, for what it's worth, is to abandon this rather futile search for an absolute way of formalizing what should be the result of induction, and just go with the latter, with all it's faults. Do engineering: use what works in some context, and keep refining it. Really, I don't even have references to give you, because I simply don't see the point of this kind of metaphysical exercise. A "necessary" attribute of, say, cheese? Come on. Why? You might read the late (not the early!) Wittgenstein for some of the arguments and problems here.

Steven Ravett Brown

back

(14) Conny asked:

Can you think conceptually without a language? If not, what implications does this have for von Humboldt's view?

---

Yes.

But I'm not going to go on about Humboldt. You do that. Look here:

Bermudez, J.L. Thinking without Words. Edited by Chalmers, D. J., Philosophy of Mind. New York, NY: Oxford University Press, 2003.

Gendlin, E. "Thinking Beyond Patterns: Body, Language, and Situations." edited by den Ouden, B. and Moen, M., 22-152. New York, NY: Peter Lang Publishing, Inc., 1991.

Grandin, T. Thinking in Pictures: And Other Reports from My Life with Autism. New York, NY: Vintage Books, 1995.

Jonker, C.M., and Treur, J. "Modelling the Dynamics of Reasoning Processes: Reasoning by Assumption." Cognitive Systems Research 4 (2003): 119-36.

and

Gibbs, R. W. Jr. The Poetics of Mind: Figurative Thought, Language, and Understanding. Cambridge, MA: Cambridge University Press, 1995.

Lazar, R.M., Marshall, R.S., Prell, G.D., and Pile-Spellman, J. "The Experience of Wernicke's Aphasia." Neurology 55 (2000): 1222-24.

Papafragou, A., Massey, C., and Gleitman, L. "Shake, Rattle, 'N' Roll: The Representation of Motion in Language and Cognition." Cognition 84 (2002): 189-219.

Reddy, M. "The Conduit Metaphor: A Case of Frame Conflict in Our Language About Language." In Metaphor and Thought, edited by Ortony, A., 164-201. Cambridge, England: Cambridge University Press, 1993.

and

Arnheim, R. Art and Visual Perception: A Psychology of the Creative Eye. Berkeley, CA: University of California Press, 1974.

Goodman, N. Languages of Art. 2nd ed. Indianapolis, IN: Hackett Publishing Company, 1976.

----. Ways of Worldmaking. 5th ed. Indianapolis, IN: Hackett Publishing Company, 1988.

Gopnik, A., Glymour, C., Sobel, D.M., Schulz, L.E., Kushnir, T., and Danks, D. "A Theory of Causal Learning in Children: Causal Maps and Bayes Nets." Psychological Review In Press (2003).

Steven Ravett Brown

back

(15) Steve asked:

I'm not sure if this is even a philosophy question, probably more of a juvenile sixth form angst question, but my question is how do you unthink, how do you unbecome the person that you are?

---

Yes, you're right, this is not a philosophy question. That's point one. Point two is that you need to go see a professional counselor, and avoid drugs. Point three is that the classic ways to get out of this kind of depression are: religion, drugs, suicide, and finding something you love doing. Well, gee, which one am I advocating? Guess. What do you enjoy, no matter how strange it seems (so long as it's not self-destructive or hurts others)? Computers? Some game? Then do it. Walking? Do it. Investing money (yes you can start with pennies)? Do it. And so forth. Point four: an absolutely marvellous way to get over depression is to exercise. Yes, strange as that seems, it works. Go run, or go lift weights, or if you must go take up racquetball or squash. Something very strenuous, and go do it frequently. And see a counselor. Now.

Steven Ravett Brown

back

(16) Dasha asked:

What comes first the egg or the chicken?

---

This is a lovely question. Lovely, but pointless. By which I mean that no answer one could ever give to such a question could possibility convey any actual information. Instead of soliciting knowledge, what this so-called question invites one to do is simply to state one's policy on the meaning of a word, namely 'chicken'. Thus, by stating that for you 'chicken' is defined by a genome bearing a specific set of DNA markers, some of which must have occurred randomly at some point in the past development of birds, it would be possible to declare 'the genome conforming egg came first, and then hatched, and then that was the first chicken, thus the egg came before the chicken'. But supposing I define 'chicken' differently, any number of answers are possible, and the point is that the whole exercise is arbitrary: there is no right way to define 'chicken' or 'egg'. Since this question comes down to how you define 'chicken', the only thing that an answer to it will tell us is what we've decided. But we know what we've decided when we decide it, so what's the point of the question? Simply to elicit a decision? But in that case why do people bother to object to an answer with a contrary answer of their own? There cannot be a contradiction between decisions: that you decide to go to the pub doesn't show that I was wrong to go to the cinema.

Perhaps, and this seems plausible, one might want to include a clause in the definition of 'chicken' that makes clear that for us the word refers to a species of bird which reproduces itself by laying eggs. This will have the result that the animal we earlier and so boldly referred to as a 'chicken' in truth came to deserve that name when it proved it's reproductive capacity, i.e., laid an egg. Thus we will say that DNA markers or no, the first chicken and first chicken egg arrived simultaneously.

Or we might say that 'egg' in this case always means something laid by a chicken, from which result we get: the chicken came first. There's no reason why someone shouldn't go that way.

But all three kinds of answer here are fundamentally uninformative — none of them tells you anything of any value whatsoever, but simply elicits a statement of policy about words, thus: 'I'd like to say that the chicken came first' etc. The question, one might say, is silly. Or in philosophical talk, it's a pretend or pseudo-question. Ask a silly question, get a silly answer. Ask a pseudo-question, get a pseudo-answer.

That, in short, is why 'which came first, the chicken or the egg?' is a byline for all silly questions everywhere, as in 'that's a chicken and egg question, isn't it? [silent smirk from down to earth type]'. Philosophers and philosophy students might do well not ask chicken and egg questions, and instead ask some more questions that matter (there are plenty of them) — or else we'll get those scientists saying that they are the only brain workers who actually get anything done.

David Robjant

back

(17) Stacey asked:

An inductive argument cannot be sound?

---

Well, if the first crow you see is black, and the second, and the third... are all crows black? What if you see a white bird that looks (except for being white, of course) and behaves just like the black birds you've been calling crows; is there anything justifying calling it a crow?

Steven Ravett Brown

back

(18) Don asked:

Where can I find something published on the following: Given that Mary is the wife of John, it follows that Mary is a wife and that John has a wife. By inspection, we see that the same applies to every statement of the form 'a = b OF c'. Thus A, OF and HAS are logical operators and A is somehow inverse to HAS. Now A is clearly element-of-a-set and OF is something close to 'function'. What is HAS? (I know that this is almost the unary function. The A relation seems to say that every value of a function is an element of the set named by the function (and that the parameter selects one of these). The HAS relation seems to say something about the potential parameters of the function).

---

It was difficult to follow your thread at the point where you declared capital 'A' to be a logical operator because this 'A' occurs nowhere in your preceding text. I'm guessing that you may mean the indefinite article 'a' capitalized or not. Now I know roughly what a logical operator is: it's the sort of thing you can draw a truth table for (And, inclusive Or, exclusive Or, If then, etc). Now there are ways of signifying the indefinite article in the language of symbolic logic (For some x... ), but that doesn't make it a logical operator as such, because you can't get together a truth table for how truth and falsity are operated on by some x, the way you can for 'and'. Look up 'truth table' in a Dictionary of Philosophy.

In any case, and just a seriously, in your rendition 'a = b of c' the '=' is a bad sign for your purpose since, we hope, there is more to Mary than the predicate 'is the wife of John'.

David Robjant

back

(19) Don asked:

Where can I find something published on the following: Given that Mary is the wife of John, it follows that Mary is a wife and that John has a wife. By inspection, we see that the same applies to every statement of the form 'a = b OF c'. Thus A, OF and HAS are logical operators and A is somehow inverse to HAS. Now A is clearly element-of-a-set and OF is something close to 'function'. What is HAS? (I know that this is almost the unary function. The A relation seems to say that every value of a function is an element of the set named by the function (and that the parameter selects one of these). The HAS relation seems to say something about the potential parameters of the function).

---

I don't know where you can find references relating to this specifically... but I know where you can ask this question with a hope of finding them. Go to this discussion group:

http://groups.yahoo.com/group/analytic/

and ask them. Someone there will probably be able to help you. If you want to consider natural language as employing "logical operators", "functions", and so forth. I think your analysis above is mistaken, myself, and that applying such operators to natural language is the equivalent of stuffing a square peg into a round hole. See my last set of answers on this site.

Steven Ravett Brown

back

(20) Selene asked:

Suppose you are inside an empty cube, what are 10 things you can see from the inside?

---

You can't see anything from the inside because ex hypothesi you are not inside. The cube is empty! (You have asked us to suppose something that cannot be supposed).

An alternative smart alec answer:

'I can see six surfaces, eight three planed corners, twelve two planed joins, myself counted as one thing, and... the lightbulb.

That makes twenty eight things. Alternatively, the light bulb is off and I see: nothing.

David Robjant

back

(21) Clare asked:

Regarding this Supervolcano. Is it going to erupt, if so estimated when and will it be big enough to kill off the entire planet?

---

Much is made of the certainty scientists have that this thing is going to blow in the next couple of thousand years or so, not excluding tomorrow. Definitely less ought to be made of this. Compare: will there be a general election this year? Definitely. Will there be a general election in the life of this dragonfly? Probably not.

David Robjant

back

(22) Cliff asked:

Is there any philosophical concept or calculus that can be used to distinguish between contingent, necessary and sufficient attributes of an object or activity? From what I have read, this is an extremely subjective exercise, whether one is dealing with structural, functional or behavioural aspects. As an — Irish — engineer, that will not do me, at all, at all. Take these two items 'X' and 'Y' (instances of class 'x' and 'y') and as different from one another as chalk is from cheese. How do I go about establishing and prioritising their attributes — from first principles?

---

Lots of philosophers have looked for the 'first principles' here. Lots — one might even, while acknowledging that many turned away from the search earlier than others, say 'all'. A notable recent attempt, much discussed in the literature for and against, was made by a chap called Saul Kripke with his 'rigid designator' theory of reference. But for myself I strongly suspect that there are no first principles here — or, which comes to the same thing, that any principles you might set up for the purpose of distinguishing essential from not essential would be your invention and tool, rather than some a priori fact which you have discovered.

I recognise the style of mind which is dissatisfied with this fact about how language depends upon our intentions, but one might more usually ascribe such a style of mind to the physicist than the engineer. Engineers, after all, are supposed to be practical. Follow what works, and don't bother with theory unless it will help in showing what works. Perhaps in Ireland the physicists are practically minded engineers and the engineers are philosophically minded physicists.

My own view would be the now completely unfashionable stance of Leibniz, to some extent echoed by Tolstoy in War and Peace, for which all of the facts about an object are essential to it.

A man who did not cross the Rubicon would not have been Caesar. A man who did not die in his bunker would not have been Hitler — and so on. A supposed difficulty for my view, and perhaps the issue which makes this quandary about language a live philosophical question of some importance, is the light this casts on the counterfactuals which are the stuff of historical analysis and science, and indeed everyday self-flagellating regret. Many philosophers start from saying that Counter-factual statements refer to the real world, and hack about with their metaphysics until they have made it conform to this peculiar starting point. But against this approach both I and Tolstoy are in agreement: there is something deeply fishy, metaphysically fishy, about 'if Napoleon had not pressed on to Moscow' and likewise about 'if only I'd turned the gas off'. The facts are what they are. Napoleon is who he is. I am the clut who forgot the gas, I am not some other person. What happen happened. What we know we know. We cannot genuinely un-know it when we formulate a counter-factual. Counter-factuals have not merely a tenuous relation to the real world, they have rather no referential relation whatsoever. They refer not to the world we live in, but to countless invented fictional worlds.

Now counterfactuals may serve some function, indeed alarmingly many diverse and conflicting functions, but these functions operate in the world of fiction, and if such counterfactuals refer at all, it is in fiction and there only. Such are the limitations of historical analysis and historians generally, likewise of economists, sociologists, and astro-physicists. I and Tolstoy are of a mind: 'if only' clauses hypothesise non-existent breaks in a fictionally staccato world, as if one could stop time for an instant, pick it up, and throw it in a new direction. Such a thing cannot be done, (as Zeno showed, says Tolstoy, as Zeno showed, says I). Time flows continuously and the water passes under the bridge. What passed passed. The known qualities of a thing are it's known qualities. All of them. If some bloke claims to be Hitler, the fact that Hitler died in 1945 is reason enough for concluding that he isn't Hitler. No fact about Hitler would be more or less important in showing that this bloke wasn't Hitler — any and all of them would serve. If it we know that Hitler did x, then we know that only the bloke who did x is Hitler. Likewise we know that a Historian who writes 'if Hitler had been admitted to the school of art' is talking nonsense, or, which is the same thing in this case, using our words to describe his own fictional world.

Now, perhaps we might like to use the name of one thing to describe another a bit like it — fine, but in such cases we should try to remember the element of fiction. This is not Hitler, but Hitler2. Perhaps from some of our counterfactual fantasies (both marxism are nationalism came from this unpromising bunch) something positive may come, such as a predictive theory that actually works for a while. In such cases of obvious success what we would other wise call a 'fantasy' we would instead call 'imagination'. But for any given fiction the chances of this kind of success occurring are slight, and the difficulty is that with the definite exception of engineering, in most areas of human 'inquiry' people will be able to go on with their fantastically derived predictive theories for generations without ever having to discover in some practical and obvious way that the picture they are pleased to call 'imaginative' is in fact mere fantasy; without ever having to discover that their favourite ideas bear no relation to reality whatsoever. Perhaps this does not matter. Or perhaps it does.

David Robjant

back

(23) Gabriel asked:

What is the purpose of life and how do we achieve it?

---

Why do you think that there is some purpose of life at all? Should there be? Perhaps there isn't! Life is not an animal nor a person. So why should there be a purpose of life? It seems plausible that only YOU define the purpose of your life!

But let's see what is implied: "Purpose" is anywhere between "goal" and "meaning". Does your life have a "goal"? Did you set such a goal for your life? Probably not. We seldom do. Or did you give meaning to your life? How is it done? A German philosopher called history-writing "giving meaning to the meaningless". Philosophy of history is mainly about the question whether history is "meaningless". The Jews and the Christians deny this by the argument, that God is the master of history. Even if we dumb humans think history to be meaningless, God knows better. He has His plans, while we do not know his plans. But Voltaire (1694-1778), one of the first "modern" historians, ridiculed this idea in his "Candide" (1759): There is no plan, history is absurd, but we humans can at least try to make the best of it. See http://kclibrary.nhmccd.edu/voltaire3.htm and http://www.literature.org/authors/voltaire/candide/. For a classical introduction to the philosophy of history see Karl Lowith "Meaning in History: The Theological Implications of the Philosophy of History" (orig. am. 1949) (http://www.amazon.com/exec/obidos/tg/detail/-/0226495558/102-2137856-4669723?v=glance)

Now apply this to your private life, which is your private history. Some people feel "guided by God" or "directed by the stars" or in other ways see their fate as determined. I know this feeling myself. But I remain sceptic on this. We always stay responsible for what we do and not. In my opinion "existentialism" and "fatalism" are likewise mistaken: Our life is our task and we have to try our best to deliver as good a work as we may.

But there are different meanings of "meaning". From the movies and novels we know the lover adoringly stammer "you give meaning to my life" to the beauty (or the beast). What does it mean? There is a beloved person who is concentrating all your thoughts and actions to the one purpose (!) of pleasing her. Thus your thoughts and actions become directed like a compass-needle by a magnet. Your life has found a purpose (!) and by this got meaning (!), because all thoughts and actions are now brought into order by this one purpose.

Of course this general model is not restricted to falling in love with somebody. Your inner compass-needle may find its magnet in God or in the arts and music, or in some research, or in a life as an explorer or an engineer etc. etc.. There are many goals.

But where is life now? Life has no purpose — YOU have. If you have some goal where to go, giving direction and meaning to your life, then your life all of a sudden has found a purpose, because you have.

But do we really always need a goal where to go, directed by a compass? What about being just fascinated by life in the way the French entomologist Jean-Henri Fabre (1823-1915) was fascinated by insects and beetles (see http://www.efabre.net/ and http://www-museum.unl.edu/research/entomology/workers/JFabre.htm)? What's the purpose of a rose, a tree, a cloud, a lion, a baby?

We like to be loved for what we are, not for what we are useful for. We don't like to be "instrumentalized". Since what will happen when be become "worthless" in old age? To think that every object and action must be "good for something" is a very modern and somewhat absurd idea, driven by the modern obsession with "progress". This was the objection of Heidegger (1889-1976) to modernity: We moderns are always too busy and too loud. We should learn to listen again to the world, since the world — the "being" — is speaking to us. But since we are pre-occupied with ourselves and with "progress", we don't hear the voice of "being". This is the essence of the story of Mary and Martha (Luke 10,38-42): Martha is busy, she wants to be useful. Mary is just listening to the words of the great rabbi. Mary knows that there are guests then and now, but this one is a really important guest, somebody to listen to. So no time to waste with being busy and "useful". We should learn to calm down, to listen, to see, to love.

All this is not "asking for a purpose". It is just "being here and now", being aware. This explains why Heidegger took Socrates to be the beginning of Occidental spiritual decline: Socrates was not listening, he was questioning and analyzing, he started the "intellectual noise" of the Occident. Because of this, Heidegger more and more admired the Pre-Socratics and then Zen-Buddhism. Zen-Buddhism could be called "the philosophy of awareness, the philosophy of listening". No ego, no purpose, just "being here and now". See http://www.rep.routledge.com/article-links/G101SECT2#S19

So what is the purpose of life and how to achieve it? Just try to open heart and mind to the world. The purpose will vanish, but the world may begin to speak to you. And that would be a real achievement.

Hubertus Fremerey

back

(24) Brian asked:

I'm reading Sartre's The Age of Reason and find so many similarities with the character Mathieu. As a result, I'm exploring my own self. In the book, Mathieu states that he is waiting, always waiting, and preparing for...something. This seems to be an excuse for not acting and living. I'm not certain the I understand the existential implication here. If he never acts, because it's impulsive and not informed, how is he free? He seems paralyzed by uncertainty.

---

First have a look at what I answered some time ago to Damon on "existentialism" in general, see Old Answers 18. There you will find much of the answer to your current question. And of course: Doing "nothing" by staying undecided is just one possible option of your liberty.

But there is another aspect: We need not always go ahead. Look at the famous figure of "Melancholia" engraved by Durer in 1514 (see http://www.princeton.edu/~his291/Jpegs/Durer_Melancolia.JPG and see http://www.wordiq.com/definition/Melancholia). Durer's "Melancholia" is surrounded by instruments of science and craftsmanship. This gives her freedom to do many things. And this exactly makes her "melancholic": "Too much freedom at hand." This was in part the idea behind the German philosopher and psychologist Erich Fromm (1900-80) writing a book under the title: "Escape from Freedom" (1941). There Fromm interpreted the German option for Hitler as such an "escape from freedom", preferring a determined "Fuhrer" to a fearful "democracy" with its many freedoms. For this is the point: If you don't decide what to make of your freedom, others will do it for you.

Sartre, himself a brainy melancholic, a modern "intellectual", tried to find a way out by activism and by permanent rebelliousness. He was unable to "build" anything of worth. Restlessness is only the other side of melancholia. If you can't build anything of worth and beauty, then you try to destroy in frustration and anger what others have built so far. Sartre admired Stalin as a man of action. For Sartre Stalin was what Hitler was for many Germans: Somebody who is moving the world and not sitting idle pondering it. This explains in part why so many "intellectuals" always fall to such "leaders": They are similar.

But this includes — on the side of the intellectuals and on that of the leaders — a "lack of love": For to love means, to care and let grow and to quietly enjoy in the way a mother enjoys a playful baby. To love means to build with loving care, to give light and life and form to what has been dead and chaotic. This is the original act of creation — an act of love. To be melancholic and undecided means to be lacking love, to shun or ignore life, to be unable to enjoy. On this see Fellini's movie "8 1/2" (http://www.imdb.com/title/tt0056801/). All great dictators and their intellectual followers and admirers are people who cannot love and therefore rage in hate and self-hate and activism — or in melancholia.

You are right: Such people are not free. They are either paralyzed or driven, which both are forms of being unfree. This was on the mind of the Apostle when he wrote (2Kor 3,17): "Where the spirit of the Lord is, there is liberty". But this is not different from what the Buddha said. To find yourself, you have to go out to the world and see and enjoy and love. Read the poems of Whitman (1813-92) on this or of his admirer Rimbaud (1854-91). To find yourself you have to forget yourself by doing some work of loving dedication. This is something like the opposite of "existentialism" and its paralyzing obsession with "the riddle of freedom".

Hubertus Fremerey

back

(25) Gail asked:

What is the philosophical stance called immaterialism

and Jeffery asked:

What is the philosophical stance of immaterialism and what does it mean to be a immaterial philosopher?

---

At face value, immaterialism is a conviction that says, there is no matter, no substance, but only form and forces. But what does it come to?

The historical background of this is a conviction, well known from Plato, that the most fundamental "reality" is not matter but "truth" — which is a "form". For what is "material" in a mathematical or logical truth, as f.i. in the "Law of Pythagoras"? Since Plato was "a mathematically thinking person" — as were Descartes and Spinoza and Leibniz — they all were deeply impressed by this fact. In their opinion, to say in the sense of materialism that mathematical truth follows from the movements of matter misses the fact that a fundamental truth is an absolute truth. A fundamental truth is not a factual truth. A factual truth about f.i. the physical nature of the solar system of course may be left to the materialist. Even the laws of nature may be "material" in this way, dependent on the fundamental structure of some universal force-fields. But this fundamental structure itself — be it superstring-theory or what else — is (in the opinion of immaterialism) not material, but spiritual, it is "immaterial form" in a way that is necessitated not by matter but by "fundamental truth". Seen in this way, the properties of matter — even of quarks and superstrings (if there are any) — are defined by immaterial, mathematical necessity, of a very beautiful (unknown) formula, and not the other way round (i.e., the formula defined by the properties of matter).

Of course: The properties of matter and of all "events" must be consistent with "fundamental truth". But the "fundamental truth" as an immaterial form is prior to all else. Immaterialism does not claim that there is no matter. It only says that matter is not fundamental, but that (immaterial) "form" is.

What about "force"? When God said according to the Bible "Let there be light" he was not using "force" explicitly. His word was taken to be "forceful" in itself. But this is religious thinking and even may be called "original superstition", attaching "mana" or "charme" or similar "magical" forces to some words and actions. In modern thinking force is always "physical and natural force" and not "supernatural force". Even "teleportation" — if it is possible at all — would be explained from "a natural force of so far unknown nature" but not from a "spiritual force". But modern physics has no clear answer to this problem: Even if you reduce electro-magnetic "forces" to the exchange of "photons", those photons must have some "interactive effect". If this effect is not "spiritual" it must be material. In any case you have to have some force to bring about anything. Thus the concept of force is in fact a deep mystery, which is not explained either by physics nor by meta-physics, but is unavoidable to explain interaction. To speak of interaction without any notion of forces is meaningless, whether in a Newtonian or in a pre-Newtonian world-view. A "causa" has to be either a material cause, a force, or an immaterial cause, a "causing necessity", which would then be a "spiritual force" by necessity.

Because of this, immaterialism is free to think of forces as "immaterial" or as "material", and of course there are philosophers in both camps.

But compare http://www.philosophypages.com/hy/4r.htm on this.

Hubertus Fremerey

back

(26) Conny asked:

Can there be any laws of nature or mathematics if you believe you get all of your ideas from experience?

---

And why does what you "believe" matter in this? The question is not whether you believe you get your ideas from experience, from goblins, the muses, the materialist dynamics of the brain, or flying saucers. The question is where you do, actually, "get" or generate your ideas. That's question #1. Question #2 is, what does "where you get your ideas" have to do with "laws of mathematics" or "laws of nature"? Is a "law of nature" something which guides or is inherent in nature, i.e., the objective world (the world which surprises us), or something we use to understand nature? Neither? Either? Both? We think that gravity acts on all matter, right? Well, is that an "idea" or is it out there in the world? Both? Neither? You need to sit down and think this through. Just what are you asking? Is your question about us, human beings, and how we understand the world, or is it about the world we're trying to understand... or something else? Is mathematics part of the "objective" world, or is it somehow separate and more basic, more pure, in some sense... and so operates with "laws" which are different in kind from natural laws? Or not?

Steven Ravett Brown

back

(27) Conny asked:

If Bohr is correct in the quantum objects may be beyond our capacity co conceptualize, then is it possible that both he and Einstein are correct, though they disagree, What do the limits of conceptualization or logic have to do with reality?

---

Come on. What are you asking here, a question about physics or a question about metaphysics/ epistemology? Make up your mind. As for both Bohr and Einstein being correct, yes, that's possible. Bohr, if you've read any of the history here, would never let himself be pinned down as to whether he thought the uncertainty in measurement reflected a real, ontological uncertainty, or merely some sort of (perhaps temporary) restriction on instrumentation and methodology, and so there is room, just barely, to conceive of Bohr's position as an instrumental one which could thus be modified or refined to coincide with Einstein's belief in what are termed "hidden variables". There's no way, however, that I'm going to go further on this. There are reams of commentary, books, etc., etc., which are readily available for you to read here. Some few of the numerous writings are:

Bohm, D. Causality and Chance in Modern Physics. New York: Harper & Brothers, 1961.

Greene, B. The Elegant Universe: Superstrings, Hidden Dimensions, and the Quest for the Ultimate Theory. New York, NY: Vintage Books, 2000.

Hess, K., and Philipp, W. "A Possible Loophole in the Theorem of Bell." Proceedings of the National Academy of Sciences USA 98, no. 25 (2001): 14224-27.

Mulhauser, G. R. "On the End of a Quantum Mechanical Romance." Psyche 2, no. 19 (1995).

Reichenbach, H. Philosophic Foundations of Quantum Mechanics. Berkeley, CA: The University of California Press, 1965.

Rowe, M.A., Kielpinski, D., Meyer, V., Sackett, C.A., Itano, W.M., Monroe, C., and Wineland, D.J. "Experimental Violation of a Bell's Inequality with Efficient Detection." Nature 409 (2001): 791-94.

And then you throw out questions on the limits of 1) conceptualization, and 2) logic?? Go read some Kant before you get into that one:

Kant, I. Critique of Judgment. Translated by Pluhar, W. S. Indianapolis, IN: Hackett Publishing Company, Inc., 1987.

----. Critique of Pure Reason. Translated by Pluhar, W.S.T. Indianapolis, IN: Hackett Publishing Company, Inc., 1996.

----. Critique of Practical Reason. Translated by Pluhar, W.S.T. Indianapolis, IN: Hackett Publishing Company, Inc., 2002.

Pluhar, in my opinion, does a very good job of translation and explanation here. There's no way I'm going to even attempt to begin answering questions like that in this forum.

Steven Ravett Brown

back

(28) Margaret asked:

"The day that a priest can show me how to transmogrify water into wine I will unlapse immediately." Any comments?

---

I would concur. In general, one can employ this general approach to the same end: If God is really as omnipotent as s/he is supposed to be, then s/he is perfectly capable of making it very obvious that s/he exists. Since s/he has so far not chosen to do so, the simplest answer as to why, is that s/he does not exist.

Stuart Burns

back

(29) Dave asked:

I find other questions and answers unsatisfactory on this subject, so I hope you don't think I'm going over a topic already dealt with.

Whenever I hear a debate about freewill, it always starts in the middle of the debate. The first thing I would like clarified is what exactly people mean by 'freewill'. I am aware that until I scrutinized it, I believed that it made sense for someone (if one accepts the idea of some kind of spiritual moral authority) to be judged on their decisions. This was due to the fact that we are free to make these decisions and thusly accountable for them. This seemed obvious.

Once I'd begun to examine this though, I discovered that what we call freewill is actually an indefinable sense we have of accountability. I don't believe that the indefinable exists. If I look in a dictionary, it defines freewill (in a philosophical sense) as to do with being free to make ones own decisions. This presupposes that the decision-making process is itself free — in which case, why is it called a 'process'? Computers run according to processes that are confined by the parameters they are set by (their circuitry and programming). We are the same. A computer can make decisions, so can we.

But this debate is old and tired and I've never been able to convince anyone of it.

After years of mulling this over, I've reduced the whole debate into an easy-to-follow and easily provable, flawless piece of reasoning. It is flawless because it is right. Here it is:

Bear in mind that I believe freewill is an illusion. The phrase does not correspond to anything except our perceptions. It is not that I think doesn't exist, it is that it can't exist. Neither us or God or anything can be free in the way that is implied by religion. Because it can't be defined, how can it be disproved? Simply by disproving the one implicit aspect of free-will: That we could be fairly held morally accountable for our actions.

Anything that takes place or can be said to have occurred must either be caused to happen or happen without cause. Neither of these allow for the notion of freewill.

I have told this to people and hoped that they would say "that is correct, therefore, although I don't know why yet, freewill must be an illusion." None has responded this way.

So, a slightly elaborated version is:

If something occurs and is caused to occur, then it only occurs because it was caused. Take away the causer and it doesn't happen (unless caused by something else), put the causer back in and it does happen.

Although, would the result be exactly the same occurrence? Arguably not, but irrelevant to the point.

If something occurs and is not caused to occur, it occurs for no reason and without cause. It is, therefore, a spontaneous random event.

We are held accountable for our actions which are occurrences.

We are either being held accountable for occurrences which are dependent on a causer or we are being held accountable for spontaneous 'random' event.

I put 'random' in speech marks because it is ambiguous. I mean random in the sense of 'without cause' i.e. anything could have taken place.

If the universe is completely causal — then we are being judged according to a chain of causality. Every single occurrence that has been part of the chain of occurrences was dependent on every link in the chain. In other words, my actions might be dependent on a raindrop 700 billion years ago. I would be judged on this.

If the universe is not at all causal, it is a completely random event and I am judged according to nothing and my actions/hopes/life are meaningless.

If it's a mixture of the two then I am being judged either according to actions dependent on a finite chain of causes (until the chain is broken by a random occurrence) or judged according to a spontaneous, uncaused event.

My question is, how can anyone possibly refute this?

---

Well, doing logic can be somewhat confusing. So for a first warm-up, I put a charming comment by Voltaire on how he came to write his "Candide" of 1759:

I must quote Lactantius, church father, who, in his chapter XIII, of his On the Wrath of God puts these words into the mouth of Epicurus:

Either god wants to remove the evil from this world, and cannot, or he can, and does not want to; or he neither wants to nor can; or he wants to and can. If he wants to but cannot, this is impotence, which is contrary to the nature of god; if he can but does not want to, this is wickedness, which is no less contrary to his nature; if he neither can nor wants to this is at once wickedness and impotence; if he wants to and can (which is the only one of these possibilities fitting for god) whence then comes the evil which is on earth?

The argument is powerful; so that Lactantius answers it very badly, saying that god wants evil but that he has given us the wisdom with which one acquires the good. It must be admitted that this answer is quite weak in comparison with the objection, for it assumes that god could create wisdom only by producing evil; besides our wisdom is pretty ridiculous!

Bolingbroke, Shaftesbury and Pope, who embodied their ideas, do not resolve the question any better than the others: their All is good means nothing more than that all is controlled by immutable laws. Who does not know that? You tell us nothing new when you observe, as all little children have done, that flies are born to be eaten by spiders, spiders by swallows, swallows by shrikes, shrikes by eagles, eagles to be killed by men, men to kill one another, and to be eaten by worms, and then, all but one in a thousand, by devils.

Here we have a clear and fixed order among every kind of animal. There is order everywhere. When a stone is formed in my bladder it is by means of admirable mechanics: calculous juices pass little by little into my blood, they filter into the kidneys, pass through the ureters, deposit themselves in my bladder, and assemble there by an excellent Newtonian attraction; the stone is formed, gets bigger, I suffer pains a thousand times worse than death, by the most elegant arrangement in the world. A surgeon, having perfected the art invented by Tubalcain, comes to thrust a sharp and cutting iron into the perineum, and takes hold of my stone with his pincers. It breaks under his efforts by a necessary mechanism; and by the same mechanism I die in frightful torments. All this is good, all this is the evident consequence of inalterable physical principles. I agree with them, and I knew it as well as you did.

If we were without feeling there would be no reason to object to this cause and effect. But this is not the point. We are inquiring whether there are any perceptible evils, and whence they come. "There are no evils," says Pope in his fourth essay on the All is good; "or if there are particular evils, they form the common good."

A strange general good! composed of the stone, the gout, all crimes, all suffering, death and damnation.

Taken from http://humanities.uchicago.edu/homes/VSA/Candide/tout.est.bien.html, which is from Voltaire, Philosophical Dictionary, edited and translated by Theodore Besterman (London and New York: Penguin Books, 1972), p. 68-74.

So much for a first check on what is logical in this world. As we all know, the ways out of this conclusion have been two: Either there is no God at all, then the question of how to vindicate God — the theodicy — vanishes. Or we call all evils a test of the faithful or a punishment of the sinners and infidels brought about by God, then this is the usual explanation given by the true believer, avoiding the inference that God is cynical. See the story of Job on this. Thus pick what pleases you most.

Now I turn to your question on freewill.

We all know, that humans behave overall quite reasonably and morally correct — more or less so. And we all know, that punishments and admonitions and models of reasonable and moral behaviour are not without value generally — more or less so. But if things work out in this way quite convincing, what are you trying to show me? Do you really try to "prove" that human behaviour is meaningless? I think we should see the argument from the opposite side: Since the logic seems to "prove" something that apparently is not true — i.e., that human behaviour is without any reasonable or moral meaning — there must be a fault in the logic, not in the reality. Thus I will look for the fault in your logic.

The core of my argument will be, that in the case of humans there is no direct connection of cause and effect, stimulus and response, in the way we may assume such a direct connection working in plants and in primitive "lower" animals. The difference of "re-acting" and "acting" seems to be denied by your approach altogether. But I think you have not proven that this can be done.

You write:

If the universe is completely causal — then we are being judged according to a chain of causality. Every single occurrence that has been part of the chain of occurrences was dependent on every link in the chain. In other words, my actions might be dependent on a raindrop 700 billion years ago. I would be judged on this.

If the universe is not at all causal, it is a completely random event and I am judged according to nothing and my actions/hopes/life are meaningless.

If it's a mixture of the two then I am being judged either according to actions dependent on a finite chain of causes (until the chain is broken by a random occurrence) or judged according to a spontaneous, uncaused event.

My question is, how can anyone possibly refute this?

Well, I will try to refute this chain of arguments — and I really do not feel like a robot obeying the hints of "a raindrop 700 million years ago". But this may be "my illusion".

Taken merely as a logical argument, your argument on first sight looks sound. But this exactly seems to be the core of the problem. Your application of the concept of causality in my opinion is too vague in several respects.

First: I think you will not invoke supernatural forces, and neither do I. So let this be agreed on.

Second: The throw of a coin or dice, while surely causal, does not allow for a prediction. Thus a random-walk, while causal, does not allow a prediction either. But this is no refutation, only a warning: Causality does not include predictability. This is interesting insofar, as "Laplace's Demon", predicting the future of the world from knowing the "initial conditions" of all atoms in the world, was a misleading creature even in the time of his invention by Laplace at around 1800 (see http://www.pha.jhu.edu/~ldb/seminar/laplace.html and see http://www.hypography.com/topics/Laplaces_Demon_112215.cfm). To know this, one does not need even Heisenberg's "Uncertainty Principle", but only classical stochastics of the random walk. Today we know of many simple "chaotic machines" that never return to the same state and thus are unpredictable. Thus if you formulate the theory of "unfree will" you could not invoke the Laplacean Demon for help, since he is dead. And he even was a primitive demon, not knowing of the pitfalls of modern Quantum Mechanics, which would kill him anyway. But you didn't invoke him. So what else could justify your concept of predictability and predetermination?

Third: As you already see from the first argument, your "raindrop" would have no effect whatsoever on any current event. It would only make some minimal "noise", while the "signals" — the real causes of the event — are very, very much louder. But what are those "signals"? Suppose while you are reading this on your computer a drop of water hits your head. If you are sitting in a closed room you would be as startled as people in a horror-movie when blood is dropping from the ceiling. Not because of the little drop of fluid, but because you KNOW that something overhead must be very wrong if it's dropping on your head or hand. Thus your horror is not "caused" by the drop of fluid, but by your knowledge of this world and of what to expect and what not.

But what you call your "experience" or your "knowledge" — which in part rests on your experience and in part on "second-hand acquired knowledge" — is a very indirect "causal agent". It's not only that surely >99% of your knowledge is second-hand or "in your genes" and not in the events themselves ("drops of fluids"). Thus the DIRECT impact of most events in your life is minimal, while the indirect impact is usually much more important.

Well, you may say that you are "pre-determined" or "pre-programmed" by your genes and your memes, i.e. culturally learned patterns of interpreting the events that hit you. But how do you prove that there is no "second order chaos" in your brain: If you are undecided what pattern of reaction to chose when coping some "event", you are in the position of somebody fighting "immoral desires". And there moral arguing steps in: When parents and teachers and friends and movies and novels and the pastor etc. all try to convince you that a certain way of acting in a certain situation is preferable to another way, they try to strengthen one possible set of options over countless other sets of options and by this "give some bias to the better choice". Seen in this way, moral education — or any advice — is meant "to break the decisional chaos".

By the principle of liberty — even according to the Bible — man should be free to choose "by insight". Since insight is generating a distance between "the event" and "the reaction", using a "lookup-table" as a sort of "delay generator", there is no direct connection between "the event" and the "answer". By this the relation of "event" and "answer" becomes unpredictable. And because you are "a learning machine", your "answer" to a certain "event" may be totally different tomorrow from what it has been today.

I think there are some problems we have to live by. While the throw of the dice is "following the laws of physics", it is — by necessity! — unpredictable. Thus while we may think that any process "following the laws of physics" must be predictable, this is apparently not the case. And this has nothing to do with "uncertainty principle" but follows from the "logic of chance": It would be meaningless to speak of a fair dice if the outcome of throwing it is predictable. Likewise the arguments against "free will" seem faultless superficially, but at the same time seem to become meaningless when scrutinized.

For what indeed are you trying to prove? On the face of it you are trying to show that there is no free will, but if asked whether you will call all humans to be mere dumb automata you will hesitate when viewing at the arguments just given. If not, then I would like to know your counter-counter-arguments. I still think this moment, that human responsibility can be justified in a consistent way.

There seems to be a trap most "determinists" fall into: To be responsible we are not required to build up our "lookup-table" defining moral and reasonable behaviour "from scratch". Most of what we do is indeed predefined from "our genes" and from "our enculturation". For this we are not responsible. But we are expected to develop some "moral, emotional, and general intelligence". This intelligence is a form of "third order distancing" from the primary event, the first being the neuronal signal, the second the "lookup-table" in our memory itself, and the third being the intelligence of making good use of the lookup-table.

Compare it to a game of chess: Firstly there are the chess-board and the figures and rules of the game, which constitute the material given. Secondly there are the wisdoms of tactics etc., which are not "given" but applicable. And thirdly there are the actual moves of the opponent. The challenge of the player is on this third level, not on the two below. In my opinion, if this "reacting on the third level" is called "pre-determined", the whole arguing becomes "supernatural" — which form of explanation was excluded from the beginning.

I still think that the core of the whole problem of free will is contained in the right interpretation of "causality". Any unqualified concept of "cause" directly runs into "bad metaphysics", avoiding all problems of predictability as shown above.

I give your introductory statement a twist, where the "I" this time is "me": "Whenever I hear a debate about free will, it always starts in the middle of the debate. The first thing I would like clarified is what exactly people mean by 'un-free will'." So please tell me how you would try to "prove" that somebody is NOT free to choose IN A MEANINGFUL WAY. This qualification is important! We never are COMPLETELY free to choose. We surely are bound by the genes and memes of our "human nature and upbringing" TO A CERTAIN DEGREE. I would never deny this and no judge would deny this either. If you are a human member of modern Western society, no judge would expect you to behave like a "little green man" from Mars. But this granted there is room left for "reasonably" responsible and rational behaviour. And this alone is the point in question here. What your argument comes to — if I understand you right — is a claim, that there is NOT THE SLIGHTEST ROOM for such a "reasonably" responsible and rational behaviour. On this you did not convince me.

But in the end I have to admit that I did not find a simple and elegant argument of refutation — if there is any. I only tried to make some difficulties with notions like "completely causal" or "not at all causal" look as difficult as indeed they are. Logical paradoxes are resolved by showing up and resolving hidden ambiguities. Cf. the famous example of the "liars paradox", where the hidden ambiguity is in the unqualified "all" of "all Cretans are liars" when the speaker is Cretan. The paradox dissolves and turns into a mere assertion of fact if the speaker is excluded or not Cretan.

I think that this whole debate on "free will" is spoilt by such a hidden unresolved ambiguity we do not understand this time. But you are right: Since philosophy is on logic and good arguing, to understand what is wrong with your "logical" conclusion is important.

But then: Perhaps we have to accept that there are many facts that can be proven "by procedure" but not "by arguments". I have no problem to imagine a man-made robot that fulfills all criteria of "free will" by using look-up-tables and good principles and learning to evaluate situations and to act meaningfully in a similar way as a chess robot. Has anybody proven so far that such a robot could be reduced to some formula? Could it be that the hidden ambiguity of the "paradox of free will" is in this FALSE assumption that it must be resolvable "by logic"?

To put it otherwise: If somebody is showing you a robot that apparently behaves reasonably in any practical respect in different situations (see the "Bicentennial Man", http://www.imdb.com/title/tt0182789/), would this not "dis-solve" the "paradox of free will" as good as any "logic" argument? Today we know of several generally accepted mathematical "proofs" that are not at all "short and elegant" but long and complicated, and even replacing formulas by procedures (cf. the "Four Colour Problem"). Perhaps we simply overestimate what logic is able to do.

And I am not speaking of Pascal's "logique du coeur" here, which is quite another matter.

Hubertus Fremerey

back

(30) Ryan asked:

Hello, I find this website very interesting, you've done a good job with it. I was asked this question for a school seminar "are there some things we can never know?" From Ryan

---

Perhaps we may never know the answer to your question. Did you think of this possibility?

But then: What do you call "a thing" — and what do you call "to know"? And who is "we"? There surely are many things around in everyday life you will never know even if you would like to — not even about yourself. Do you know yourself? And we probably will never know what innumerable dead people said or thought in their lifetime, including Neanderthals and Cromagnons. Only if after death we find out to live on in some spiritual body in a very different world where every thought of every person and animal etc. becomes readable and knowable, this could be proven wrong. But for the moment we don't know whether such a world does exist at all.

To know something you first must be aware of something and this something should be meaningful. Thus if you think there is no God, then the question of His plans or His grace will not disturb you as it disturbed Augustinus and Luther, and the question whether you could know this becomes meaningless. But if you want to find out whether Nessie does indeed exist, you could (in principle) isolate and empty the whole of Loch Ness and see what is left. It is not done because it is too costly and the tourist industry may suffer from the outcome.

In mathematics and logics, if a problem looks unsolvable, it is generally "bad formulated". Thus the impossibility of "squaring the circle" has been proven only some 120 years back, when it was better understood "what the true nature of the problem was." Likewise "Fermat's Last Theorem", while looking quite simple, has been proven only some ten years ago using the most advanced mathematical methods and insights. But this does not prove that there will never be a mathematical problem that resists all efforts to solve it. Perhaps we humans are too stupid to find the right approach to solve it. And even if there are computers some day which are brighter than any human mathematician, this too would not prove that all problems could be either solved or shown to be badly posed.

And then: How does a cow solve mathematical problems? It doesn't, because the cow has not the brains to do math. So how do we know that we are not "cows" when compared to some "higher intelligences" we do not know this time, but which may be living elsewhere in the universe or in the multiverse or in some future? Since your question referred to what "we" can never know, this difference is meaningful: What "we" cannot know may very well be knowable to "higher intelligences" — if there are any. But who knows?

And there even may be the following situation: To know, you have to see some pattern against some background, some signal against the "noise" of the unknown. But we often don't see such a pattern even if it is just below our nose. This is a daily experience of all school-kids and teachers in the world. Now the world is like an open book, lying just below the nose of mankind. Using science and philosophy and theology and experience and computers and whatever "we" try to see and understand the patterns in this great book, but we may just be too stupid and miss most — and perhaps the most important — messages written there before our eyes.

So how do we know that we know? I think that we do know not very much, but of course I can't prove it as long as I don't know what can be known. The usual experience is: The more you know, the more you feel ignorant, because one aspect of knowing is to see many problems popping up before your mind you never imagined before. If you think you know much, it's only because you are a naive rookie.

Hubertus Fremerey

back

(31) Markary asked:

What is the imperative and benefit of dualism in our daily lives. How does the fact that mind and body are separate help us?

---

One answer might be that as mind and body are separate, this permits the possibility of mind [or soul] continuing to exist after the demise of the body. In a Christian context, the chances of the soul obtaining an eternal life are increased by the following the Ten Commandments and performing the works of God in our daily, mortal lives. Hence this is one example of the importance and benefit of dualism in our lives.

Martin Jenkins

back

(32) Manousakis asked:

How do J.S. Mill and Karl Marx differ in their conceptions of social progress?

---

John Stuart Mill

John Stuart Mill proposes that patriots have overthrown tyrants so that they themselves could govern democratically [e.g. Revolutions in France and America]. Unfortunately another tyranny now arises — the tyranny of the majority. Freedom of Opinion and Individuality ought to be recognised and defended against this tyranny. The development and progress of humanity depends upon these two sources. Existing opinions and modes of life ought to be subject to criticism and alternative modes of life: this will guarantee human progress and avoid stagnation.

Karl Marx

Karl Marx argued that history is characterised by the class struggle. The economic progress of capitalism [which Marx pays tribute to in the beginning of The Communist Manifesto] is conditional on it benefiting the ruling class in the form of profit. Means and methods of Production do not develop for their own sake, they develop on condition that they are profitable for the ruling class. Society is accordingly organised to meet profit and not needs. For the proletariat, social progress is dependent upon capitalism being overthrown. This would usher in world communist society where profit would be replaced by social need. Poverty, Inequality, exploitation, war and social antagonism would become things of the past.

Differences

Marx saw social progress as being enshrined in collectivism. Mill saw progress safeguarded in and developing from sovereign individuals although he edged toward a type of socialism later on. Marx arguably worked in a teleological framework of historical change [Slavery to Feudalism to Capitalism to Communism]. Mill does not really invoke anything 'metaphysical'. Whilst Marx invokes a clash between real social classes as vehicles of progress, Mill favours ideas as such vehicles. Mill defended private enterprise as derivative of individual liberty whilst Marx saw private enterprise as a fetter on the progress of individual liberty

Martin Jenkins

back

(33) Sadi asked:

Why did peoples perception of the world change after the 1889 Paris world fair? how did the new inventions of the beginning of the 20th century have an impact on people?

---

The first "modern" World Fair was in London in 1851 (there have been some other expositions before, see http://www.wordiq.com/definition/List_of_world%27s_fairs. The United Kingdom was on the way to become "the fabric of the world". London was perhaps the largest and most bustling city in the world then, and by introducing the new steam-engine in 1776, which had been modernized by James Watt from older, much less efficient models, the modern industrial era took off and fabrics and the surrounding agglomerations of workers housings began to sprout everywhere. The steam engine was put to work in the new steam-ships (Fulton from 1805) and in the new railways (Stephenson from 1825). Besides that the steam-engine allowed for mass-production and for a much better mining of coal and ore, which were needed for steel-production. There was much progress in chemistry, and electricity was practically "invented" by the work of Faraday and Maxwell. You should look up all those names in the internet or in a lexicon (f.i. enter http://www.wordiq.com/definition/Michael_Faraday, http://www.wordiq.com/definition/James_Clerk_Maxwell, http://www.wordiq.com/definition/Robert_Stephenson, http://www.wordiq.com/definition/Robert_Fulton, http://www.wordiq.com/definition/James_Watt)! These and many other links are free. Thus learn to use the net to your advantage and look up "Industrial Revolution" (http://www.wordiq.com/search/industrial%20revolution)! And look up "innovation" (http://www.wordiq.com/definition/Innovation).

This Industrial Revolution was connected to a social revolution by the 'rise of the working class', which was unknown before. Thus the famous 'workers parties' and 'socialist parties' were formed in the second part of the 19th century together with the new fabrication plants, coal- and steel-regions and industrial townships. See http://www.wordiq.com/definition/Social_Democrats and do some clicking there.

Thus you see that the whole society in Europe and in the USA was changing under the impact of 'Industrial Revolution', where the World Fair in Paris of 1889 (to honour the 100th anniversary of the French Revolution of 1789!) was only one of many similar events. F.i., the Americans had honoured in 1876 the 100th anniversary of the Declaration of Independence by a World Fair in Philadelphia, where the DoI had been formulated and published in 1776.(see http://www.wordiq.com/definition/Centennial_Exposition and http://libwww.library.phila.gov/CenCol/) In both World Fairs (there were many more!) the strongly expanding new industry wanted to show proudly its new strength and to open new markets where to sell steam-engines, electrical engines, locomotives, cranes, harvesting machines, etc., but many smaller practical machines of everyday use too.

What changed the minds of people was a new outlook into the future, a new optimism of progress for the masses. When the Eiffel-Tower was built to be the symbol of the Paris World Fair of 1889, the French 'Impressionist' (Monet, Manet, Degas, etc.) were flowering in Paris. And Monet did several paintings of the new railways. (see http://www.ibiblio.org/wm/paint/auth/monet/ and http://www.ibiblio.org/wm/paint/auth/monet/paris/monet.st-lazare.jpg)

But of course, this same progress was felt as a decline by many other people, mainly by the old Elites who feared 'the rise of the masses'. The First World War (in its time simply 'The Great War' since nobody imagined a second one) was not the cause of the general change of society but its consequence. It was the final 'clash of cultures' — of the new socialist masses and the old conservative bourgeoise and nobility. Afterwards, in 1918, most kings and emperors and other princes stepped down, while socialist and liberal parties took over.

This is the general background, in which to place what you call 'the new inventions' — the railway, the steamship, the car, the telephone, the airplane and many others. While the steamship was from 1805, the famous great ocean-liners of the 'Titanic'-type were built in the years between 1895 and 1915, while the railways became a mass-vehicle for everybody from about the 1860s. Together with the telephone and a much more efficient post-service all these innovations stand for a new era of mass-travel and mass-communication. To this add the new mass-media — the daily newspapers 'for the boulevard', the books and monthlies, the photos. The time of the cinema and the airplane was more after the War.

You see how all these new inventions created the modern mass-culture you are used to. The real problem today is to imagine how life has been before in the time of NO telephone, NO cars, NO railways, NO electricity, NO radios, NO photos etc.. In the years of Goethe (1749-1832) and Jefferson (1743-1826) none of these were available. It has been said that a Roman of the times of Caesar, when transferred by a time-machine into the time of Goethe around 1800, would have had not much trouble to get along and find himself at home there, but Goethe himself, when advanced by the same time-machine into the year 1930 would have felt completely at a loss and misplaced. This is the surface of it.

The deeper problem was a clash of optimisms against pessimisms, of people — mainly from the working classes — who saw a grand hope and progress, and others — mainly from the governing classes — who saw a grand decline and the breakdown of all order as it was known to them. Never before in all of human history has there been a comparable transformation of the way of living and of the general character of the social order in such a short time.

But there was a deeper conflict too, that I only can hint at: Thoughtful people saw modern industry and its scientific and technical resources and the hope of a general progress and wealth as a moral and spiritual danger of utmost importance. When people began to move from a life near to nature in the countryside to a life in the 'sinful' cities and the workplaces and offices there, they would become 'alienated' — from nature, from God, from their own soul, from their neighbour. They would turn from pious to arrogant, from frugal to greedy people, and the arrogance and greed would render them indifferent to any deeper thoughts and values and feelings. This was the great concern not only of Marx, but also of Nietzsche and Heidegger and Marcuse and many others. Thus it was not only the ressentiment of the old ruling class against the new working class that made modernization and the rise of the affluent society look like a program of dubious and debatable value. There was — and in fact even is — a widespread fear that industrialization and modernization may turn out to be a great disaster for mankind. Well, I don't share this pessimism, but you should at least see the problem.

Now you see how your little question / Why did people's perception of the world change after the 1889 Paris world fair? How did the new inventions of the beginning of the 20th century have an impact on people?/ leads to very large answers if taken seriously.

Hubertus Fremerey

back

(34) Stacey asked:

An inductive argument cannot be sound?

---

No, it cannot, but this does not necessarily mean that it is wrong or useless. Why? The standard example is: If you used to think by 'an inductive argument' that all swans are white, then go to the zoo to see the black swans. Only if you understand the necessity of some fact you have proven it. Thus the 'law of gravitation' is a true 'law of physics' and not a mere generalization from some observed facts. But, by the way: Even "laws of physics" may be wrong! They too have to be checked then and now by new experiments and new insights. There is not much knowledge — perhaps even no knowledge at all — that we can prove to be absolutely sound.

Induction is always a form of guessing. It's like saying: 'Since it has happened this way up to now, it may happen this way always'. While we cannot be sure on this as long as we don't understand some underlying principle explaining WHY it has happened this way up to now, the guessing is not bad, even if it rests on only one observation. Indeed there was only ONE example to prove Kepler's laws of planetary motion — the orbit of Mars — but this one was sufficient for Newton to "prove" his law of gravitation.

Why is inductive reasoning reasonable even if not sound? Life is a practical task. We cannot have theories on everything. We all live on TRUST all day long. Even to sit down on a chair or eating ones meal is betting on that the chair is real and that the meal is not poisoned. We are living in a regular world — mostly. People who are overanxious and hesitant always without some good cause are not fit for life. Pessimists generally are not wiser than optimists, they are only less trusting. This is not the same as 'check it, and check it again, and check it once more.' To be careful can be very important — f.i., when checking a technical device like an airplane. But this is not the same as being anxious all the time. We have to risk a part of our life then and now — when entering a training or a job or a marriage or buying a house etc.. In each of these cases we can make a mistake, even a severe one. But even then we try to trust in some rule of experience, thus arguing 'by induction'.

This explains why people get mad and anxious if there is too much change around. All of a sudden many rules of experience don't apply and people feel insecure and fall to 'fear of freedom' and call for some dictator who imposes 'law and order' again. Our world should be reliable and trustworthy as much as possible. Only then can we make plans and go ahead. This explains why lying, pretending, cheating and perjuring are severely punished generally. They tend to destroy the fundaments of mutual trust needed in social life. And this explains why learning from error is most effective most times: To be harmed by error or stupidity is a punishment and costly. Insurers take great pains to find out what went wrong in a disaster and how to prevent it next time.

Thus in a more or less regular and rule-guided, in a stable and trust-worthy world, trusting in the value of 'merely inductive' arguments is not the worst tactical principle — even while it is not sound in the strictest sense. And it does not imply carelessness.

Hubertus Fremerey

back

(35) Cassie asked:

Does philosophy exist in today's society?

---

I must confess that I am not quite sure where this question is coming from! Are we asking whether there is a sense of philosophy in society? Do people in general philosophize? Has philosophy any value in today's society?

As philosophy can be interpreted as the love of wisdom or the love of truth, it seems likely that the subject plays an important part in many lives. Many people will be philosophizing without realizing that they are involved in the practice. Those concerned about morals, politics, religion and human behaviour in general, are in effect involving themselves in philosophy.

The impact of science and technology on society, the manipulation of the education system by governments and big business are all central to the notion of philosophy in society. In fact, any concerns in society open to debate or different points of view are fair game for the philosophical approach. The current concerns with global warming also have a philosophical dimension as well as a scientific one. Rampant capitalism and the destruction of natural habitats goes much further than straight-forward ecological concerns. What about abortion, genetic engineering, the global increase in poverty and starvation? Are these not concerns for philosophy?

If you are asking whether society in general finds any interest in academic philosophy, it never has!!

John Brandon

back

(36) Elaine Grace asked:

What are the reasons for greediness? Why do they acquire such an attitude?

---

Greediness is usually linked to selfishness. A biological excuse is self preservation as seen in birds and animals in general. Where food may be in limited supply eating as fast as possible ensures an adequate intake, perhaps our ancestors acquired the same habits. However, in humans this seems to have extended into the competitive aspects of society, particularly the financial domain. The more affluent societies have become, the greater has become the urge in some to grab as much as they can in as little time as possible, and the more they get the more powerful they become and with it the more opportunity to grab even more. A greedy person must of necessity embrace the philosophy, "to hell with you Jack, I'm alright". Not a good platform for any society to establish itself on.

John Brandon

back

(37) Peter asked:

What is the meaning of the term "bewusseinslage"? I was unable to find the answer on the web and even on German search engines. Yet I found it used in a philosophical context: 1. Chapter One ... speculative questions concerning the elements of mind, the nature of conscious content (for example, imageless thought, attitudes, Bewusseinslage, etc.) that I ... http://members.aol.com/wutsamada/chapter1.html [Found on Google, Yahoo!, Ask Jeeves] Thanks, Peter

What you want is the word "Bewusstsein", which means, roughly, "consciousness". Go here:

http://dict.tu-chemnitz.de/

and play with it.

Steven Ravett Brown

back

(38) Barbara asked:

What is the metaphysical meaning of being bitten by a bat?

---

That you've been bitten by a bat.

Steven Ravett Brown

back

(39) Jamel asked:

How does an ethical philosopher go about answering the question concerning what is really right and what is really wrong.

---

With great trepidation, I hope.

Steven Ravett Brown

back

(40) Pavan asked:

Why don't we have bliss all the time? and why should we suffer?

---

Why not? Why shouldn't we be in pain all the time? Why should we feel anything at all? What assumptions about the world are behind your question, and what justifies them?

Steven Ravett Brown

back

(41) Leon asked:

Why do you think people who take drugs ie heroin knowing the suffering they have to go threw to get that fix and know all the pain there going to endure but still do it its like chopping your own arm off and not really giving a f--k what im trying to say do you think that a drug addict is the definition of madness?

---

Yes you would, wouldn't you. Take a look at the article here:

http://www.sciencentral.com/articles/view.php3?type=article&article_id=218392511

on smoking for some insight into just how bad the problem is.

Steven Ravett Brown

back

(42) Mokarram asked:

As I am preparing my paper, have come across some passages in some Marxists documents which are difficult to me to understand. I need a philosopher versed in Marxism to answer my problems. Thanks in advance.

---

There once was a Marxist who bought
Several books of his favorite thought
They diminished his wealth
And yet stayed on his shelf
Which is par for the course for his sort

David Robjant

back

(43) Al asked:

If one follows the naturalistic materialistic reductionist view of consciousness and your brain is exactly copied by a computer would the computer be YOU or a copy of YOU? if your body could switch between using your computer brain and your organic brain 'they would be indistinguishable from one another but I still suspect if the organic brain died only a copy of you would remain. But I think people like Daniel Dennett claim it would be the same person - is this what he thinks?

---

'If...'? If!

David Robjant

back

(44) James asked:

What are some Hindu philosophical ideas that similar to Plato's?

---

Too many! Although it is hard, one might more honestly say impossible, to guess what is a case of a distantly shared linguistic history (Nietzsche has something aggravating to say on this) and what is a case of positive philosophical agreement supported by more recent intellectual contact, say between Hindu thinkers/traders and the presocratics. Whenever we get positive evidence about this, we seem to find that people underestimate the degree to which people moved around in the ancient world, but the problem there is: we don't often get evidence.

Several difficult cases of obvious parallels with mysterious ramifications in one single quote:

Bhagavad Gita 18/61 "And [God's] power of wonder moves all things — puppets in a play of shadows — whirling them onwards on the stream of time"

For puppetry read the Plato's central allegory of the Cave in the Republic — Plato talks about the objects carried along the road and appearing over the wall to cast shadows into the cave, but the way he does this is intended to evoke stages and puppetry — an under remarked aspect of the allegory, under remarked because there is so much going on there, what with the fire, the sun, shadows, copies and all.

For shadows (very important in Plato) — see same allegory.

For the stream of time (flux, very important to Plato's attitude towards empirical knowledge): see countless references to Heraclitus in Plato and also his gnomic remark (it slips me where) that Chronos (time divinity) is the offspring of Oceanus and Tethlys (water gods).

See, also, the way Plato talks about Pleasure and Pain in the Republic, as involving illusion: strong resemblances with Hindu thought and Buddhist developments of it here.

Again, Plato's degrees of reality thesis — the picture of levels of reality and of our slow and difficult moral ascendance through them might easily have been lifted from a Sanskrit faith. (Though I make no positive claim that they were).

Again, Plato allows, at least mythically, a kind of reincarnation: we exist in the world of the forms before birth, the soul is immortal in some important sense, also see the Myth of Er.

I don't mean to do an impression of that character on Goodness Gracious Me! and declare "That Plato? Indian!", but a surprisingly cogent case can be made for likemindedness. In many cases Plato is addressing indian questions and providing Hindu or Buddhist-like answers. There's certainly a sense in which various Sanskrit religious/philosophical thinkers are doing the same kind of thing as Plato, where A.Y.Ayer (for instance) isn't.

David Robjant

back

(45) Todd asked:

Hello, my question is based on this passage from an old book I found at a church basement sale: "A man's theory of life is largely a matter of temperament or constitution. He may find support for it in the teachings of philosophy, but he is apt to choose a philosophy which suits his way of thinking rather than to let his views of life be determined by abstract philosophic teachings." Frank Frost Abbott, The Common People of Ancient Rome. NY: Scribner's, 1911. So, for example, I think that living a life modeled on what I know about Stoicism is a good idea — but I am susceptible to the romantic notion of Socrates or Epicurus living an ascetic life because of my own need for identity. How do I know if I have chosen a sound philosophy to implement or if Stoicism appeals because it makes me feel superior? Is there a way to test the absolute value of a philosophy versus emotional convenience? Suggestions for further reading are most welcome. Thanks from Madison, Wisconsin.

---

In answering this question I'd probably begin along these lines:

'Is there a foolproof way to test the absolute value of a philosophy versus it's emotional convenience? No. For some fairly easy to comprehend emotional reasons, people have been trying to come up with such a method for hundreds of years. See, in this bag: Aristotle, Descartes, Wittgenstein (both times). Abbott, however, goes too far. I would say that the power of ones constitution to act on one's philosophy rather implies the reciprocal power of one's philosophy to act on one's constitution. For as the former power shows, they are of like substance.

At any rate, Abbott has no real evidence for his affected pessimism about intellect. The thought merely appeals to him — presumably because it offers him a windy note of importance whilst saving him the trouble of any further thought about any philosophical view. Well, that shows the character of the man!

By the way, I treat your question as remote from the question "is there absolute truth?". What you are asking is: can we altogether divorce the value of this picture of the truth (a 'philosophy') from it's appeal to us? To that question the answer is 'no'. But the fact that we abandon this picture for that better one manifests the pull of a distant goal. I'd say that the ideal of perfection is necessary and present in the idea of improvement. We'd say that the better picture was more truthful, and there, already, would be the absolute.'

But then I'd think better of this and add:

'A better picture, a more truthful one, would acknowledge the very important truth that 'convenience' is not a simple concept, and that there are different kinds of convenience, even different kinds of 'emotional convenience'. For one thing, what is convenient for an apprentice is not what is convenient for a master carpenter. And in the the evolution of both ability and taste here we can see the distant pull of something, we might say a standard outside convenience — this would be the more usual way of saying this in moral language. Or — and this is important — we might put the very same point in quite a different register of moral or aesthetic language, and say that since that distant pull is an emotional one, at any rate acts on the emotions, this too is part of the way we evaluate the convenience of the thing. Is this chair convenient? It pleases in all kinds of ways. But something niggles — perhaps the angle of the neck support — it is not perfect. Something cack-handed about the execution, the limitations of it's maker stamped upon it. Only perfection, we might say, could be perfectly convenient. And that way of going on with the word 'convenient' would serve to remind us of American Pragmatists and of Plato's distant Good, in the light of which we see and feel everything. For what we are really talking about here, in the language of pragmatist philosophers, is 'taste'. And I'm with Dewey here: so far from there being no disputing about taste, taste is in fact the one thing that all disputes are at bottom, and rightly, about. What is the good life? That is a question of taste and convenience. But because we can quite immediately see that what is convenient for someone with no taste or ability is not perfectly convenient, and because we can conceive of improvement in skill (eg in chair making) and taste (eg in wine), the importance of convenience does not banish the absolute but, on the contrary, makes the Good the supreme matter of philosophical inquiry.'

Having said that I'd probably conclude: 'near enough for present purposes' and desist.

David Robjant

back

(46) Reinette asked:

I am an electronic engineering student and in a course on philosophy especially for engineers, I have been asked what the problems of observation are. I am to give some examples from laboratory work and experiments which I have done. How do I know how to limit my essay briefly introduced to. Are there certain definite problems with observation listed?

---

Imagine a net. The net falls on the world. We can then say something about each and every square. Like, here there is a fish, here there isn't. A problem of observation: the available observations depend upon the net used. For an observation isn't an observation here unless expressed through the net. A net with squares extending over hundreds of kilometers would not be much use for many purposes. We would often speak of a smaller mesh as yielding a more accurate description. A further problem of observation: there can in principle be no such thing as a smallest net, so to what are these 'more accurate' nets approximating?

There are many other things to think about. In essence the problem of observation is the problem of language. Our scientific measures are the net. Language is the net. How does language refer to the world? Can it capture everything? Can it capture anything? How much does it depend upon us? How much is it arbitrary? Can it be perfected? Is there a perfect language? What makes this language superior to that? And so on. There are a variety of conflicting views about such matters, all with their merits and arguments. You ask, in essence, for an introduction to western philosophy. There is much to read. And no way of indicating a neatly separated area which will be digested and done away with in short order. There is no local 'problem of observation', I fear. There isn't even a neatly distinct area called 'the philosophy of science'. But, anyway, one good book in the philosophy of science is The Structure of Scientific Revolutions by Kuhn. I imagine that you are pressed by a deadline and would like to avoid reading if at all possible. Still, it would be no bad thing if you read it, essay or no essay. And, differently, you might enjoy and be sparked by A Y Ayer's Language Truth and Logic, well written piffle though it is.

David Robjant

back

(47) Mani asked:

I have the following question. Does time exist in nature? The way we know time is through the changes and the way we know changes is comparison of a event before with the event now. Which means if we don't have memory, we cannot determine what was before to compare with. This means that without memory there is no time. Or is it true to say memory creates the time. Your answer on this will be more helpful. Thanks, Mani.

---

'This means that without memory there is no time'. No it doesn't.

To begin with I can simply reject your opening claim that the way we know time is through changes. Arguments in favour of such a claim prove to be very peculiar and slippery on close examination, even when they aren't stated in German, and in any case you don't offer any of them. I would say, against the arguments that you fail to offer, that change can't be a signal for the passage of time, since if it were that would imply that change is something we passively perceive. And I reject the thought that change is something we passively perceive on the grounds that for there to be change there must first be a thing which changes, and I would say that thinghood is something we do: seeing this as one thing and that as another takes more positive intellectual capacity than that possessed by a worm. Since the perception of both thinghood and change is active and not passive, it follows, I say, that there is time before there is any change, time, I say, in which we are active. Activity implies time. Our activity at the bottom of thingyness and change implies the existence of an extended time in which to act, and that remains so even when, with a failing memory, our activity begins to go awry, misfire, and eventually wane. That we were active once is enough to show the necessary pre-existence of time. To put this floridly, time did not grow out of us, we grew up in time. Which is just a pretty way of saying what common sense says: that our parents and countless generations lived before us, and that when we die the sun will continue to shine. No?

David Robjant

back

(48) Bill asked:

I have a friend who is often judging and labeling other people. Although I find it irritating, I wonder if by telling her (or just believing) that she is 'judgmental,' that I would, myself, be behaving hypocritically?

---

Bill, what a neat knot.

I find fantastically irritating the sort of person who hands down confident comments on people they don't really want to get to know, or holds forth about lessons that should be learned from this case and lessons to be learned from that, when they have only the faintest idea what they are talking about, and where the rigour with which they scrutinise their sources of information mimics the parrot. Often, I think to myself, they are not so much trying to picture and categorize the world, as to picture and categorize themselves. It's no so much an outward thing as an inward one. Such gossips delight in confidently asserting this and that, showing to themselves how they are not as silly as so-and-so, much more savvy than sally, and so on. Of course, the need to go on about the imagined failings of others in order to boost ones own opinion of oneself is the most fantastic sign of one's own insecurity and general pointless nobodiness. It is fantastically ugly. People who are really ugly, and I don't mean noses, do this sort of thing — and they usually pause for you to join. The way they react if you demur can sometimes show how much they needed your support in their disguised ego-massage. The distaste of non-gossips for gossips is as nothing to the wounded anger felt by gossips for non-gossips.

Gossiping, of course, features pretty big as a sin (false witness, and these days egoism is rightly considered a sin), and nothing I've said here would be out of place in a sermon (apologies all). So I've not stinted on the judging here. But, judging aside, have I been 'judgmental'?

I guess 'judgmental' is one of those made up words got up for the purpose of cunning confusion. "You are being judgmental!" someone says, and the only thing one can be absolutely sure about is that they want you to stop. But do they want you to stop being judgmental, or to stop judging, or do they think the two come to the same thing ?Hard to tell — although what we can see, and you've pointed this out, is that if to be judgmental is just the same thing as to judge, then one can't accuse someone of being judgmental without being judgmental oneself. Neat, huh? I like the way this made up word sets us off into a relativist spin even as our mouths are forming the terminal '...mental'.

However, perhaps there's a way out of this mish mash: a way to fix a clear meaning for 'judgmental' and work with that. When we're annoyed at the sort of character who is "often" judging and labeling other people, and we reach for the condemnatory term "judgmental", I don't think that it's really this particular judgment or that particular judgment that we're objecting to. The problem isn't that they made a judgment about someone. The problem is the way they are at it nearly all the time — the way we start to see a habit and a sort of fixation emerge — a fixation which when it becomes clear makes us think differently about each particular judgment that they make. And the lines on which I at least start to think differently are the ones I set out above: here is someone obsessed about their own status and continually harping on about other people's imagined failings in order to pump themselves up. That's the sort of person I'd call "judgmental". And the irony of it is, as I said, that they are not really judging at all. It's just a kind of disguised egoism talk. It has nothing to do with looking at the world or trying to be accurate or knowing people or knowing some secret about how to live. Now, if I were to make a judgment about such a person, if I were to meet one, and thank God and my good luck and my thirty one years that I don't know any obvious examples just now, and if I was to think about such a person and let the word 'judgmental' form in my head as a judgment about them, I'd be judging, sure enough, but I wouldn't be being 'judgmental' because I wouldn't be reaching this verdict as part of a diseased habit of displaced egomania.

Or would I?

Well, there I'd just have to keep a watch.

David Robjant

back

(49) Maria asked:

Is freedom overrated? can you export freedom and democracy like Bush thinks? what value do philosophers like rousseau and Hobbes put on the concept of freedom?

---

Freedom is particularly underrated by those who can't remember not having it. Whilst one may have an interesting philosophical argument (with reference to Rousseau etc) about the extent to which it is possible to force people to be free, it is rather odd that commentators assume President Bush has a view on this deep matter. What he says he believes is more simply a) that the majority of the people in question happen to want democracy and b) that it is possible to use force to remove the impediments to this desire for democracy, and further c) that only the use of force would have achieved this in the cases of Afghanistan and Iraq. Some popular discussion about c) aside, it would be difficult to deny that Bush is being proved right on counts a) and b).

David Robjant

back

(50) Todd asked:

Hello, my question is based on this passage from an old book I found at a church basement sale: "A man's theory of life is largely a matter of temperament or constitution. He may find support for it in the teachings of philosophy, but he is apt to choose a philosophy which suits his way of thinking rather than to let his views of life be determined by abstract philosophic teachings." Frank Frost Abbott, The Common People of Ancient Rome. NY: Scribner's, 1911. So, for example, I think that living a life modeled on what I know about Stoicism is a good idea — but I am susceptible to the romantic notion of Socrates or Epicurus living an ascetic life because of my own need for identity. How do I know if I have chosen a sound philosophy to implement or if Stoicism appeals because it makes me feel superior? Is there a way to test the absolute value of a philosophy versus emotional convenience? Suggestions for further reading are most welcome. Thanks from Madison, Wisconsin.

---

Very interesting question Todd. In fact we could say that most of us would tend to find specific philosophical points of view or systems that can support our own 'way of life', and then later on, call it our choice. First of all, philosophical systems are systems of ideas that are generated in a determined time period. You couldn't be a stoic as some greeks were stoics, only try to adapt the stoic system of ideas to modern day times. For instance, what could be called hedonistic in ancient Greece could be called stoic now. But this is a parallel discussion. For what it concerns your question, in my view, the only way you could be sure of 'the absolute value of a philosophy versus emotional convenience', as you put it, is by way of first living according to a set of fixed parameters, and then, later on, trying to see if a system exists defending such a life style. I don't agree that you can choose a philosophy so you can implement it. Sometimes we call our trademark behaviour our philosophy of life (at least in my country we do so), and that is because it's like we develop our own ideas on how to life and conduct our life, and some of us, later on, study philosophy and see that someone has thought things trough, and actually has built a coherent system of ideas that encompass our own. You probably won't be an original, but you never know...

Nuno Hipolito

back

(51) Mani asked:

Does time exist in nature? The way we know time is through the changes and the way we know changes is comparison of a event before with the event now. Which means if we don't have memory, we cannot determine what was before to compare with. This means that without memory there is no time. Or is it true to say memory creates the time. Your answer on this will be more helpful. Thanks, Mani.

---

This is a hard one to answer. I think that without memory there is no identity. Part of our identity relates to change, and we know time by change. Doesn't mean that without memory there is no time, just means that it could exist but we wouldn't be able to know it. Change happens even when you are not observing it, but you relate it to time, to better understand it. We could accept a reality of 'states' instead of 'now and after'. You could say that an object is in 'state x' instead of 'better', or 'brighter'. I think it's narrow-minded to consider that Nature just evolves and goes through changes. In fact we see this more clearly in humans: instead of seeing a human older, why don't we say that a human is in a different state, that he is no longer the same human, he is a different one, a different person all together. You could say that the space between states is change. But I disagree. Change demands comparison, and you cannot compare states, it would be like comparing apples and lemons.

Nuno Hipolito

back

(52) Kasha asked:

God gives life and only God can take it away do you agree give reasons for your answer showing that you have considered more than one point of view.

---

If you believe in God, you probably see Him as the source of all life. So God is a giver of life and it is only logical that the giver of life should be the one with the authority to take it. The greatest sin is to take life, and in part that has to do with religious and sociological reasons. On the other hand, if you don't believe in God, you probably think that life evolved without divine intervention, so there is no one that can claim to have the right to take life as he or it pleases. My personal view is that no one should have absolute rights over another life, be that someone God, a parent or the State. Even if God did give life, as a parent does, that doesn't mean that it gives that someone the right to dispose of that life as he or it pleases. In an extreme analysis, existentialism defends that we are born free from God, and we are condemned to be free, to make our own choices. Eventually, your last most import choice is to live, and how to live. Most modern Stated condemn suicide and euthanasia to keep social order, and prevent a chaotic situation in which people would be free to decide their own fate, and not what is best for the stability of the society around them.

Nuno Hipolito

back

(53) Cal asked:

Is it morally justifiable that some are born rich and others poor? Explain?

---

Nature has no moral actions. Only human beings can have moral or immoral actions, and you do not choose to be born rich or poor.

Nuno Hipolito

back

(54) Al asked:

If one follows the naturalistic materialistic reductionist view of consciousness and your brain is exactly copied by a computer would the computer be YOU or a copy of YOU? if your body could switch between using your computer brain and your organic brain 'they would be indistinguishable from one another but I still suspect if the organic brain died only a copy of you would remain. But I think people like Daniel Dennett claim it would be the same person- is this what he thinks?

---

I thought about this once too, because of teleportation theory. I always thought that if you are teleported you die, and then an exact copy of you is created elsewhere. Star Trek never was never quite the same after this. One thing is for sure: if you make a copy, even an exact copy, it's still a copy, not the original. Microsoft feels quite strongly about this, from what I've heard.

Nuno Hipolito

back

(55) Emma asked:

I am writing an inclusion essay and concentrating on a member of the class that is always absent on a Monday, how do people feel about either, holding the whole class back to accommodate the one child or carrying on work and excluding that child? please voice your opinion.

---

I know this is a far fetched idea, but did you consider giving him notes regarding the work he misses on Mondays, so simultaneously he could catch up and the work could progress?

Nuno Hipolito

back

(56) Bill asked:

I have a friend who is often judging and labeling other people. Although I find it irritating, I wonder if by telling her (or just believing) that she is 'judgmental,' that I would, myself, be behaving hypocritically?

---

The key word in your question is 'often'. If you do it often, yes, you would be acting the same as your friend, but if you do it just once, no, you're just pointing thing out for her. You can't be considered judgmental if you just do it once in a while, for the 'right reasons'. Of course, if you do it once, but for the wrong reasons — you are angry at her, or you are masking your own will to criticize others — then you are acting in a hypocritical way.

Nuno Hipolito

Someone asked:

What is a hippy and do you have any facts about them?

---

From what I know, a hippie is someone that lived in the 1960s and believed in peace and love. There are no clear facts about them. All data from that time period is incoherent because most people recording facts at that time were also hippies.

Nuno Hipolito

back

(57) Tershia asked:

I am writing a paper for school and I need some guidance. The paper I have to write is "What do philosophers think or say about GOD?" Where am I to find some information on the website regarding your comments?

---

Philosophers say lots of things about God. But the most interesting things they say, in my opinion, are things related to His existence. You can find information on the web about this just by searching with the key phrase 'proofs for the existence of god', or similar.

I can also refer you to this two excellent books:

Abbagnano, Nicola; Dictionary of Philosophy

Scribano, Emanuela; L'existence de Dieu, Editions du Seuil

Nuno Hipolito

back

(58) Larry asked:

I think someone asked this but why is the concept of odd and even a philosophical illusion? I just couldn't find the answer on your site but still why? My math, history and english teachers are all stumbled to the question and they all know philosophy, they want to know too! So?

---

You could try your... philosophy teacher. I think 'odd and even' is a philosophical idea. We classified numbers in different classes, so we could better use them, in math and other sciences. A prime number isn't a 'better' number; it just has 'different qualities' from most other numbers. It's the same with an odd or an even number. For instance, if you sum up two odd numbers you get an even number, that's a fact. Sum up two even numbers, and you don't get an odd one. Why is that? Odd and even numbers must be different then. Understanding something is all about simplifying: we differentiate numbers, so we can better see where they can be used. The fact is we don't know if the numbers really are odd and even, we just call them that to better understand them. In this way, our use of language and of a theoretical idea isn't an illusion, just a way to simplify and understand.

Nuno Hipolito

back

(59) Leon asked:

Why do you think people who take drugs ie heroin knowing the suffering they have to go threw to get that fix and know all the pain there going to endure but still do it its like chopping your own arm off and not really giving a fu--k what im trying to say do you think that a drug addict is the definition of madness?

---

I know from personal experience, and you probably know too, that you can't say to someone that something hurts x amount. People just don't believe you, and they need to try it themselves. Parents know this too well, because all they do is warn their children against the dangers of life out there. Kids still need to screw to understand later on what they meant. Even if you see the effects of drug, you don't experience them yourself, so it's kind of a non-real thing to you, kind like a video game driving simulator, compared with real life driving. A drug addict isn't mad, he's probably just desperate. And when you're desperate, you stop at nothing to feel better. You could call a friend and ask for help, or you could blame the world, do drugs, etc... sometimes this choice isn't that clear. Desperation is overwhelming and you become less rational, and you try to find escapes, not considering the consequences, you just want to feel better right now, at whatever the cost.

Nuno Hipolito

back

(60) Todd asked:

Hello, my question is based on this passage from an old book I found at a church basement sale: "A man's theory of life is largely a matter of temperament or constitution. He may find support for it in the teachings of philosophy, but he is apt to choose a philosophy which suits his way of thinking rather than to let his views of life be determined by abstract philosophic teachings." Frank Frost Abbott, The Common People of Ancient Rome. NY: Scribner's, 1911. So, for example, I think that living a life modeled on what I know about Stoicism is a good idea — but I am susceptible to the romantic notion of Socrates or Epicurus living an ascetic life because of my own need for identity. How do I know if I have chosen a sound philosophy to implement or if Stoicism appeals because it makes me feel superior? Is there a way to test the absolute value of a philosophy versus emotional convenience? Suggestions for further reading are most welcome. Thanks from Madison, Wisconsin.

---

In one sense this is true. People often do choose their overall theory of life by choosing something which fits in with their own prejudices but these people are philosophical tourists not philosophers.

The true philosopher is only interested in what is true and it is this search for the truth and willingness to accept it no matter where it leads us that distinguishes the true philosopher from the tourist. Philosophy is an attempt to rise above our prejudices.

Shaun Williamson

back

(61) Gerald asked:

What if any are there similarities and differences between philosophy and critical thinking? Is critical thinking a prerequisite for philosophy?

---

Critical thinking is merely a method or way of thinking which can be applied to any subject e.g. economics, literary criticism, mathematics etc. However critical thinking is essential for philosophy so it is a prerequisite. I won't try to define the subject matter of philosophy since I don't think it can't be summed up in any simple formula but philosophy is more than critical thinking.

Shaun Williamson

back

(62) Mani asked:

I have the following question. Does time exist in nature? The way we know time is through the changes and the way we know changes is comparison of a event before with the event now. Which means if we don't have memory, we cannot determine what was before to compare with. This means that without memory there is no time. Or is it true to say memory creates the time. Your answer on this will be more helpful. Thanks, Mani.

---

Well I don't think my answer on this will be helpful. Certainly time exists and so does the colour red. But that is only to say that the word 'time' exists in the English language and has a use. Sentences like 'memory creates time' may have a poetic use i.e. they conjure up a picture but of course memory creates nothing and it doesn't write letters either. You may need memory in order to have a sense of time and having a clock will also enable you to keep track of time. If I am blind then I cannot understand what words like 'red' mean in the same way as a sighted person does but this doesn't mean that the colour red doesn't exist.

Shaun Williamson

back

(63) Hamisu asked:

Write in your own view in not more than five pages that if you don't learn from history, you are doomed to repeat it. answers should be in relevance with the social sciences awareness.

---

This isn't a philosophical question. It is an attempt to get someone to write your school essay for you. Philosophers are far too busy to do that unless you offer them lots of money.

Shaun Williamson

back

(64) Mokarram asked:

As I am preparing my paper, have come across some passages in some Marxists documents which are difficult to me to understand. I need a philosopher versed in Marxism to answer my problems. Thanks in advance.

---

We don't know what your problems are because you don't explain what it is that you don't understand. So we can't help you.

Shaun Williamson

back

(65) Tony asked:

Are the questions asked here answered by philosophers, or by philosophy professors?

---

All philosophy professors are philosophers. Some philosophers are not professors. So I suppose the answer is that all the questions are answered by philosophers and some of these philosophers are professors and some of them are not professors.

Shaun Williamson

back

(66) Barbara asked:

What is the metaphysical meaning of being bitten by a bat? ---

Sorry you have lost me here. There is no metaphysical meaning to this. However some bats carry rabies so if you are bitten by one go to a doctor.

Shaun Williamson

back

(67) Christian asked:

Who is the first philosopher in the world?

---

I am assuming that this question is asking for the first individual whose works have approached philosophical topics. In a sense, all of humankind has their own philosophy. This includes those first humans that came in inhabit the world in some way or another. However, the first documents of philosophy come from India. The famous Hindu texts, "the Vedas", and can be dated somewhere around 1400 B.C.E. These texts espoused the belief in an ultimate reality known as Brahman. Several differing theories would emerge later to elaborate the the Vedic conception of God. Of particular interest advaita (non-dualistic) theory of Hinduism, accredited to Sankara, and the visistadvadvaita (modified non-dualistic) theory, accredited to Ramanuja. Sankara believed that Brahman was accessible on two levels: 1.) the highest level known as nirguna Brahman; 2.) the lower level known as saguna Brahman. The highest level is incapable of being expressed any human knowledge or language because it is completely other than the universe. On the lower level, humankind's expression of this ultimate reality is seen as a projection of God that is ultimately flawed by humankind's fallible nature; thus, all gods are merely flawed projections of the infinite "More". Ramanuja disagreed with such a notion and insisted that all of the gods expressed by humankind find their place within Brahman. Since the Vedas are not ascribed to any particular thinker and since Sankara and Ramanuja came much later, I will also explain who the first identified philosopher was.

The first greek philosopher was Thales (624-546 B.C.E.). He came to believe that the world was surrounded and composed of water. He also adopted the first known beliefs that have become identified with naturalism. However, before him came many that we know very little: Lao Tzu, Chuang Tzu, Confucius, Nagarjuna, and a host of Jewish philosophers (plus many, many more). You might also like to take a look at those that came after Thales, mainly Anaximander, Pythagoras, Heraclitus, Democritus, and Protagoras. After all of these came the great Socrates. The answer was probably more than you were bargaining for, but it should provide you with some information for any research you may conduct.

Robert Lovvorn

back

(68) Todd asked:

Hello, my question is based on this passage from an old book I found at a church basement sale: "A man's theory of life is largely a matter of temperament or constitution. He may find support for it in the teachings of philosophy, but he is apt to choose a philosophy which suits his way of thinking rather than to let his views of life be determined by abstract philosophic teachings." Frank Frost Abbott, The Common People of Ancient Rome. NY: Scribner's, 1911. So, for example, I think that living a life modeled on what I know about Stoicism is a good idea — but I am susceptible to the romantic notion of Socrates or Epicurus living an ascetic life because of my own need for identity. How do I know if I have chosen a sound philosophy to implement or if Stoicism appeals because it makes me feel superior? Is there a way to test the absolute value of a philosophy versus emotional convenience? Suggestions for further reading are most welcome. Thanks from Madison, Wisconsin.

---

This is a nice statement of an extremely nasty problem, and I'm glad to see that it was published, and horrified to find that it was ignored, in 1911. And there are more precedents' for it. But here's the problem. You can want to be guided by faith, i.e., a belief in something which is backed up, ultimately, by not more than a feeling of enthusiasm or something similar; or you can choose to be guided by reason. That latter sounds, good, right? The Enlightenment, Western philosophy, etc., etc... Aristotle... But then you realize that "reason" has to be based on something, some postulates, hypotheses, or whatever you want to call them, and how do you accept those? Faith? Um... Reason? Um. Right. So then what? Well, you can do what that book says one does, i.e., choose the basis of your rationality by some feeling you have... and this is what most people do... and we're back to faith, in effect, aren't we.

What else can you do? In a sense, nothing. You have to start somewhere. But that answer neglects the basis of empiricism, which is one reason that I, here in this forum, and other places, make such a point of citing empirical studies. What am I talking about? I'm talking about the dynamics of empirical-theoretical interactions: their evolution through time. So here's what you do: you take some stance, based, most likely, on some feeling you have. You think about it, you work out it's implications... and then you test those implications against, yes, the real world. That stuff out there. And you do that testing in as honest a fashion as you can... i.e., being prepared to fail. To have your precious system fall down. Be "wrong", yes. And then what do you do, when the inevitable happens, and you're wrong? Do you spend the rest of your life trying to prop it up, defending it to the death against all challengers...? Like most people? Well, no. Not if you're honest, which precious few are. What you do is, yes, that horrible thing: change your ideas in accordance with the results of the test(s).

What I'm describing here is the reason that science, when done properly, is unique in human endeavors. There are no hypotheses which are beyond testing against reality in the scientific paradigm (yes, yes, including that one), and reality, as best we can observe it, is the touchstone against which theory must succeed or fail. I'm talking about something like Bayesian statistics here, where one starts with a "prior" and uses further results to modify that. If done correctly (and if it's possible to apply such a procedure to the particular investigation), a Bayesian procedure will inevitably converge, eventually, to the correct answer.

Now the sticking point here is of course whether it's possible, and if so how, to apply such procedures to something for which they're not designed, e.g., "pure", "philosophical" thinking (yes. whatever that is). I know of no systematic way to do that... but look at Kitcher, below ("The advancement..."). Sorry to leave you hanging on this... but the only way I know, generally, is just to attempt to remain alert and flexible, and keep employing data to correct one's hypotheses. Over and over.

The recent literature on the intimate interplay of emotion on rationality (and vice-versa... which in my opinion should be studied more than it has been so far) is enormous... you might start here:

Damasio, A.R. Descartes' Error; Emotion, Reason, and the Human Brain. New York, NY: Avon Books, 1994.

Lynn, F. M. "The Interplay of Science and Values in Assessing and Regulating Environmental Risks." Science Technology & Human Values 11, no. 1, Spring (1986): 40-50.

Lynn's article is a real eye-opener.

Then... here you go...

Bechara, A., Damasio, H., and Damasio, A.R. "Role of the Amygdala in Decision-Making." Annals of the New York Academy of Science 985 (2003): 356-69.

Blasi, A. "Emotions and Moral Motivation." Journal for the Theory of Social Behaviour 29, no. 1 (1999): 1-19.

Delplanque, S., Lavoie, M.E., Hot, P., Silvert, L., and Sequeira, H. "Modulation of Cognitive Processing by Emotional Valence Studied through Event-Related Potentials in Humans." Neuroscience Letters 356 (2004): 1-4.

Dolan, R.J., and Vuilleumier, P. "Amygdala Automaticity in Emotional Processing." Annals of the New York Academy of Science 985 (2003): 384-55.

Fields, C. "The Role of Aesthetics in Problem Solving: Some Observations and a Manifesto." Journal of Experimental & Theoretical Artificial Intelligence 16, no. 1 (2004): 41-55.

Greenspan, P. "Emotional Strategies and Rationality." Ethics 110 (2000): 469-87.

----. "Emotions, Rationality, and Mind/Body." Philosophy 52, no. Supp (2003): 113-25.

Lane, R.D., and Nadel, L., eds. Cognitive Neuroscience of Emotion. Edited by Davidson, R. J., Ekman, P. and Scherer, K.R., Series in Affective Science. New York, NY: Oxford University Press, 2000.

LeDoux, J.E. "Emotion Circuits in the Brain." Annual Review of Neuroscience 23 (2000): 155-84.

Lerner, J.S., and Keltner, D. "Beyond Valence: Toward a Model of Emotion-Specific Influences on Judgement and Choice." Cognition and Emotion 14 (2000): 473-93.

McGaugh, J.L. "The Amygdala Modulates the Consolidation of Memories of Emotionally Arousing Experiences." Annual Review of Neuroscience 27 (2004): 1-28.

Miller, E.K., and Cohen, J. D. "An Integrative Theory of Prefrontal Cortex Function." Annual Review of Neuroscience 24 (2001): 167-202.

Murphy, S. T., and Zajonc, R. B. "Affect, Cognition, and Awareness: Affective Priming with Optimal and Suboptimal Stimulus Exposures." Journal of Personality and Social Psychology 64 (1993): 723-39.

Parvizi, J., and Damasio, A.R. "Consciousness and the Brainstem." Cognition 79 (2001): 135-59.

Rolls, E.T. "The Orbitofrontal Cortex and Reward." Cerebral Cortex 10 (2000): 284--94.

Scherer, K.R. "The Neuropsychology of Emotion." In Psychological Models of Emotion, edited by Borod, J., 137-62. New York, NY: Oxford University Press, 2000.

Shaw, P., Lawrence, E., Baron-Cohen, S., and David, A.S. "Role of the Amygdala in Social Sensitivity." Annals of the New York Academy of Science 985 (2003): 508-10.

Smith, C.A., and Ellsworth, P.C. "Patterns of Cognitive Appraisal in Emotion." Journal of Personality and Social Psychology 48, no. 4 (1985): 813-38.

Van Gelder, T. "What Might Cognition Be, If Not Computation?" The Journal of Philosophy 92, no. 7 (1995): 345-81.

Varela, F. J., and Depraz, N. At the Source of Time: Valance and the Constitutional Dynamics of Affect [http]. [cited 2004]. Available from http://www.liane.net/arobase.

Wilson, F.A.W., and Ma, Y-Y. "Reinforcement-Related Neurons in the Primate Basal Forebrain Respond to the Learned Significance of Task Events Rather Than to the Hedonic Attributes of Reward." Cognitive Brain Research 19 (2004): 74-81.

Zelazo, P.D. "The Development of Conscious Control in Childhood." Trends in Cognitive Sciences 8, no. 1 (2004): 12-17.

On Bayesianism... well, I'm not a statistician... you can glance at these, and any basic text on it will do, really (though Sander Greenland has some meta-statistical articles on it, as does David Freedman):

Freedman, D., and Humphreys, P. "Are There Algorithms That Discover Causal Structure?" Synthese 121 (1999): 29-54.

Gopnik, A., Glymour, C., Sobel, D.M., Schulz, L.E., Kushnir, T., and Danks, D. "A Theory of Causal Learning in Children: Causal Maps and Bayes Nets." Psychological Review In Press (2003).

Greenland, S. "Induction Versus Popper: Substance Versus Semantics." International Journal of Epidemiology 27 (1998): 543-48.

Koehler, D.J., White, C.M., and Grondin, R. "An Evidential Support Accumulation Model of Subjective Probability." Cognitive Psychology 46 (2003): 152-97.

Shiffrin, R. M. "Modeling Memory and Perception." Cognitive Science 27 (2003): 341-78.

Kahneman, D., and Miller, D.T. "Norm Theory: Comparing Reality to Its Alternatives." Psychological Review 93, no. 2 (1986): 136-53.

And generally,

Goldman, A. "Epistemology and the Evidential Status of Introspective Reports." Journal of Consciousness Studies 11, no. 7-8 (2004): 1-16.

Jack, A.I., and Roepstorff, A. "Why Trust the Subject?" Journal of Consciousness Studies 10, no. 9-10 (2003): v-xx.

Kitcher, P. "The Naturalists Return." The Philosophical Review 101, no. 1 (1992): 53-114.

----. The Advancement of Science; Science without Legend, Objectivity without Illusions. New York, NY: Oxford University Press, 1993.

----. "On the Explanatory Role of Correspondence Truth." Philosophy and Phenomenological Research LXIV, no. 2 (2002): 346-64.

Kleiner, S.A. "Explanatory Coherence and Empirical Adequacy: The Problem of Abduction, and the Justification of Evolutionary Models." Biology and Philosophy 18 (2003): 513-27.

Kuhn, T. S. The Structure of Scientific Revolutions. 3rd ed. Chicago, IL: The University of Chicago Press, 1996.

Narens, L. "The Irony of Measurement by Subjective Estimations." Journal of Mathematical Psychology 46 (2002): 769-88.

Piaget, J. Judgment and Reasoning in the Child. Translated by Warden, M. Edited by Ogden, C. K., International Library of Psychology, Philosophy and Scientific Method. Paterson, NJ: Littlefield, Adams & Co., 1959.

Popper, K. R. The Logic of Scientific Discovery. Translated by Popper, K. R., Freed, J. and Freed, L. English, 1958 ed. New York, NY: Harper & Row, 1968.

Sagan, C. The Demon-Haunted World: Science as a Candle in the Dark. New York, NY: Random House, Inc., 1996.

Steven Ravett Brown

back

(69) Son Ho asked:

Explain to me the parapsychology argument and how does it works.

---

I can't, since there is none, and what little there is, doesn't work. Look here:

Blanke, O., and Arzy, S. "The out-of-Body Experience: Disturbed Self-Processing at the Temporo-Parietal Junction." The Neuroscientist 11, no. 1 (2005): 16-24.

Blanke, O., Mohr, C., Michel, C.M., Pascual-Leone, A., Brugger, P., Seeck, M., Landis, T., and Thut, G. "Linking out-of-Body Experience and Self Processing to Mental Own-Body Imagery at the Temporoparietal Junction

." The Journal of Neuroscience 25, no. 3 (2005): 550-57.

Giovannoli, J. The Biology of Belief: How Our Biology Biases Our Beliefs and Perceptions: Rosetta Press, Inc., 2000.

Hines, T. Pseudoscience and the Paranormal: A Critical Examination of the Evidence. Buffalo, NY: Prometheus Books, 1988.

Langdon, R., and Coltheart, M. "The Cognitive Neuropsychology of Delusions." Mind & Language 15, no. 1 (2000): 184-218.

Sagan, C. The Demon-Haunted World: Science as a Candle in the Dark. New York, NY: Random House, Inc., 1996.

Schick, T., Jr., and Vaughn, L. How to Think About Weird Things: Critical Thinking for a New Age. Mountain View, CA: Mayfield Publishing Company, 1995.

Shermer, M. Why People Believe Weird Things: Pseudoscience, Superstition, and Other Confusions of Our Time. New York: W. H. Freeman and Co., 1997.

Young, A.W. "Wondrous Strange: The Neuropsychology of Abnormal Beliefs." Mind & Language 15, no. 1 (2000): 47-73.

Steven Ravett Brown

back

(70) Leon asked:

Why do you think people who take drugs ie heroin knowing the suffering they have to go threw to get that fix and know all the pain there going to endure but still do it its like chopping your own arm off and not really giving a f--k what im trying to say do you think that a drug addict is the definition of madness?

---

This is a complex question with no simple answers. Most of the people who become addicted to drugs like heroin are young (teenagers). When you are that age you are often willing to experiment. You may have been told by adults that heroin is addictive and will ruin your life but at that age you don't necessarily believe adults. So for example all teenagers know that some day they will die but although they know this rationally they don't believe emotionally. So young car drivers are more likely to speed, cause accidents and believe that they are better drivers than they are in reality. Young people are also more superstitious than older people so they are often prepared to believe that there are some magic answers to the problems of life. If you can try to see the movie 'Drugstore Cowboy' which is an intelligent movie about drug use and why people are attracted to drugs.

Shaun Williamson

back

(71) Larry asked:

I think someone asked this but why is the concept of odd and even a philosophical illusion? I just couldn't find the answer on your site but still why? My math, history and english teachers are all stumbled to the question and they all know philosophy, they want to know too! So?

---

Well I don't believe that the concept of odd and even is a philosophical illusion. An even number is divisible by two and this can be really useful if you want to divide something into two equal halves. So the only question is who said that 'the concept of odd and even is a philosophical illusion'. Unfortunately I don't know.

Shaun Williamson

back

(72) Mani asked:

I have the following question. Does time exist in nature? The way we know time is through the changes and the way we know changes is comparison of a event before with the event now. Which means if we don't have memory, we cannot determine what was before to compare with. This means that without memory there is no time. Or is it true to say memory creates the time. Your answer on this will be more helpful. Thanks, Mani.

---

You're touching on an enormous subject here, one which is still being debated. Think about it this way... one way to look at it is that memory, in effect, creates time... or at least the sense or feeling of temporality. But what is memory, without time? That is, of course memories are co-present with one's experiences so in that sense they are not "in" the past... but since they are "of" the past, there must have been a past, in some sense, and so there is an objective "time" in some sense. There's a book out recently about the implications for time of relativity theory, on Godel's formulation of that concept (A World Without Time: The Forgotten Legacy Of Godel And Einstein, by Palle Yourgrau). But that's only one point of view. There are many others, from Aristotle through Kant and on up through the process philosophers (e.g., Whitehead and Taylor). Furthermore, the debate is much more complex when we think of the way we must conceptualize time, i.e., as spatial. We are "in" time; time "flows"... and so forth. Well, just how metaphorical are these conceptions, and how objective?

A very very few refs from the enormous literature on this:

Capek, M. The Philosophical Impact of Contemporary Physics. New York, NY: Van Nostrand Company, 1961.

Crowell, S. G. "Metaphysics, Metontology, and the End of Being and Time." Philosophy and Phenomenological Research LX, no. 2 (2000): 307-32.

Dartnall, T. "Does the World Leak into the Mind? Active Externalism, 'Internalism' and Epistemology." Cognitive Science 29 (2005): 135-43.

Dummett, M. "Is Time a Continuum of Instants?" Philosophy 75 (2000): 497-515.

Fernandez-Duque, D., Grossi, G., Thornton, I.M., and Neville, H.J. "Representation of Change: Separate Electrophysiological Markers of Attention, Awareness, and Implicit Processing." Journal of Cognitive Neuroscience 15, no. 4 (2003): 491-507.

Gallagher, S. "Suggestions Towards a Revision of Husserl's Phenomenology of Time-Consciousness." Man and World 12 (1979): 445-64.

----. "Sync-Ing in the Stream of Experience: Time-Consciousness in Broad, Husserl, and Dainton." Psyche 90, no. 10 (2003).

Gentner, D., Imai, M., and Boroditsky, L. "As Time Goes By: Evidence for Two Systems in Processing Space->Time Metaphors." Language and Cognitive Processes 17, no. 5 (2002): 537-65.

Glicksohn, J. "Temporal Cognition and the Phenomenology of Time: A Multiplicative Function for Apparent Duration." Consciousness and Cognition 10 (2001): 1-25.

Husserl, E. On the Phenomenology of the Consciousness of Internal Time. Translated by Brough, J.B. Edited by Bernet, R. Vol. IV, Edmund Husserl: Collected Works. Dordrecht, The Netherlands: Kluwer Academic Publishers, 1990.

Johnson, J.E., and Petzel, T.P. "Temporal Orientation and Time Estimation in Chronic Schizophrenics." Journal of Clinical Psychology 27, no. 2 (1971): 194-96.

Kikyo, H., Ohki, K., and Sekihara, K. "Temporal Characterization of Memory Retrieval Processes: An Fmri Study of the 'Tip-of-the-Tongue' Phenomenon." European Journal of Neuroscience 14 (2001): 887-92.

Lehmann, H.E. "Time and Psychopathology." Annals of the New York Academy of Sciences 138, no. 2 (1967): 798-821.

Read, R. "Is 'What Is Time?' a Good Question to Ask?" Philosophy 77 (2002): 193-210.

Reichenbach, H. The Philosophy of Space and Time. Translated by Reichenbach, M. and Freund, J. New York, NY: Dover Publications, 1958.

Robbins, S.E. "Semantics, Experience and Time." Cognitive Systems Research 3 (2002): 301-37.

Taylor, C. Human Agency and Language; Philosophical Papers, Volume 1. 8th ed. New York: Cambridge University Press, 1996.

Zelazo, P.D., and Sommerville, J.A. "Levels of Consciousness of the Self in Time." In The Self in Time: Developmental Perspectives, edited by Moore, C. and Lemmon, K., 229-52. Mahway, NJ: Lawrence Erlbaum Associates, 2001.

Steven Ravett Brown

back

(73) Pamela asked:

What is the meaning and purpose of our life if the ultimate end is death?

---

What has the meaning and purpose of life got to do with death? Are you saying that because the Pope died his life had no meaning or purpose? I would advise that you consider your own life and decide on its meaning and purpose. I feel sure that your family and friends will be able to point out the value of your life to themselves. It is a selfish attitude to believe that we live for ourselves alone, and that our lives have no meaning for others. In this context I refer you back to the late Pope. What happens after death? I have no idea; and may I say, neither has anyone else.

John Brandon

back

(74) Alex asked:

What is faith?

---

A good question, and not easily answered. Some might say that faith is a special sort of belief. But how can that be? Is it possible to grade belief? Does it make a difference if I say, "I believe it will rain tomorrow", as opposed to, "I have faith that it will rain tomorrow"? It might if in the first case I have not heard the weather forecast, but in the second case I have. In the second case I seem to have something on which to base my prediction. This seems to be the case in the context where the term 'faith' is an essential part of the language, in other words, religion. Consider Christianity for instance. Simple belief in the revealed wisdom of Christianity does not seem sufficient, it is therefore alleged that the statements in the literature of the New Testament and the Church give sufficient evidence for faith in Jesus, rather than simple belief. As we have seen in history , faith is something to die for, belief is not. Like beauty, faith is in the eye of the beholder! Also, as John Wesley made clear, faith has to be worked at, belief has not.

John Brandon

back

(75) Someone asked:

What is Aristotle's full name?

---

The normal way of differentiating between individuals having some fairly common given name was by the patronym; thus Aristotle was 'Aristoteles Nicomachou.' Diogenes Laertius V.1 gives the mother's name and his native city as well, 'Aristoteles Nicomachou kai Phaistidos Stageirites.' This resembles the Spanish practice of naming by conjoining paternal and maternal family names.

James Dye